Programs & Examples On #Greenspunning

How to get the user input in Java?

Here is how you can get the keyboard inputs:

Scanner scanner = new Scanner (System.in);
System.out.print("Enter your name");  
name = scanner.next(); // Get what the user types.

Starting a shell in the Docker Alpine container

Usually, an Alpine Linux image doesn't contain bash, Instead you can use /bin/ash, /bin/sh, ash or only sh.

/bin/ash

docker run -it --rm alpine /bin/ash

/bin/sh

docker run -it --rm alpine /bin/sh

ash

docker run -it --rm alpine ash

sh

docker run -it --rm alpine sh

I hope this information helps you.

How can I detect if this dictionary key exists in C#?

Here is a little something I cooked up today. Seems to work for me. Basically you override the Add method in your base namespace to do a check and then call the base's Add method in order to actually add it. Hope this works for you

using System;
using System.Collections.Generic;
using System.Collections;

namespace Main
{
    internal partial class Dictionary<TKey, TValue> : System.Collections.Generic.Dictionary<TKey, TValue>
    {
        internal new virtual void Add(TKey key, TValue value)
        {   
            if (!base.ContainsKey(key))
            {
                base.Add(key, value);
            }
        }
    }

    internal partial class List<T> : System.Collections.Generic.List<T>
    {
        internal new virtual void Add(T item)
        {
            if (!base.Contains(item))
            {
                base.Add(item);
            }
        }
    }

    public class Program
    {
        public static void Main()
        {
            Dictionary<int, string> dic = new Dictionary<int, string>();
            dic.Add(1,"b");
            dic.Add(1,"a");
            dic.Add(2,"c");
            dic.Add(1, "b");
            dic.Add(1, "a");
            dic.Add(2, "c");

            string val = "";
            dic.TryGetValue(1, out val);

            Console.WriteLine(val);
            Console.WriteLine(dic.Count.ToString());


            List<string> lst = new List<string>();
            lst.Add("b");
            lst.Add("a");
            lst.Add("c");
            lst.Add("b");
            lst.Add("a");
            lst.Add("c");

            Console.WriteLine(lst[2]);
            Console.WriteLine(lst.Count.ToString());
        }
    }
}

Initialise numpy array of unknown length

You can do this:

a = np.array([])
for x in y:
    a = np.append(a, x)

java.sql.SQLException Parameter index out of range (1 > number of parameters, which is 0)

This is an issue with the jdbc Driver version. I had this issue when I was using mysql-connector-java-commercial-5.0.3-bin.jar but when I changed to a later driver version mysql-connector-java-5.1.22.jar, the issue was fixed.

Confirmation dialog on ng-click - AngularJS

You don't want to use terminal: false since that's what's blocking the processing of inside the button. Instead, in your link clear the attr.ngClick to prevent the default behavior.

http://plnkr.co/edit/EySy8wpeQ02UHGPBAIvg?p=preview

app.directive('ngConfirmClick', [
  function() {
    return {
      priority: 1,
      link: function(scope, element, attr) {
        var msg = attr.ngConfirmClick || "Are you sure?";
        var clickAction = attr.ngClick;
        attr.ngClick = "";
        element.bind('click', function(event) {
          if (window.confirm(msg)) {
            scope.$eval(clickAction)
          }
        });
      }
    };
  }
]);

The HTTP request is unauthorized with client authentication scheme 'Negotiate'. The authentication header received from the server was 'NTLM'

Not this exact problem, but this is the top result when googling for almost the exact same error:

If you see this problem calling a WCF Service hosted on the same machine, you may need to populate the BackConnectionHostNames registry key

  1. In regedit, locate and then click the following registry subkey: HKEY_LOCAL_MACHINE\SYSTEM\CurrentControlSet\Control\Lsa\MSV1_0
  2. Right-click MSV1_0, point to New, and then click Multi-String Value.
  3. In the Name column, type BackConnectionHostNames, and then press ENTER.
  4. Right-click BackConnectionHostNames, and then click Modify. In the Value data box, type the CNAME or the DNS alias, that is used for the local shares on the computer, and then click OK.
    • Type each host name on a separate line.

See Calling WCF service hosted in IIS on the same machine as client throws authentication error for details.

Count number of records returned by group by

How about:

SELECT count(column_1)
FROM
    (SELECT * FROM temptable
    GROUP BY column_1, column_2, column_3, column_4) AS Records

Add an incremental number in a field in INSERT INTO SELECT query in SQL Server

You can use the row_number() function for this.

INSERT INTO PM_Ingrediants_Arrangements_Temp(AdminID, ArrangementID, IngrediantID, Sequence)
    SELECT @AdminID, @ArrangementID, PM_Ingrediants.ID,
            row_number() over (order by (select NULL))
    FROM PM_Ingrediants 
    WHERE PM_Ingrediants.ID IN (SELECT ID FROM GetIDsTableFromIDsList(@IngrediantsIDs)
                             )

If you want to start with the maximum already in the table then do:

INSERT INTO PM_Ingrediants_Arrangements_Temp(AdminID, ArrangementID, IngrediantID, Sequence)
    SELECT @AdminID, @ArrangementID, PM_Ingrediants.ID,
           coalesce(const.maxs, 0) + row_number() over (order by (select NULL))
    FROM PM_Ingrediants cross join
         (select max(sequence) as maxs from PM_Ingrediants_Arrangement_Temp) const
    WHERE PM_Ingrediants.ID IN (SELECT ID FROM GetIDsTableFromIDsList(@IngrediantsIDs)
                             )

Finally, you can just make the sequence column an auto-incrementing identity column. This saves the need to increment it each time:

create table PM_Ingrediants_Arrangement_Temp ( . . .
    sequence int identity(1, 1) -- and might consider making this a primary key too
    . . .
)

how to set imageview src?

To set image cource in imageview you can use any of the following ways. First confirm your image is present in which format.

If you have image in the form of bitmap then use

imageview.setImageBitmap(bm);

If you have image in the form of drawable then use

imageview.setImageDrawable(drawable);

If you have image in your resource example if image is present in drawable folder then use

imageview.setImageResource(R.drawable.image);

If you have path of image then use

imageview.setImageURI(Uri.parse("pathofimage"));

symfony 2 No route found for "GET /"

The above answers are wrong, respectively aren't answering why you're having troubles viewing the demo-content prod-mode.

Here's the correct answer: clear your "prod"-cache:

php app/console cache:clear --env prod

MS-access reports - The search key was not found in any record - on save

Another potential cause for this error is Sandbox Mode, which prevents MS Access from running certain statements that are considered unsafe. This can be disabled by setting the following registry key...

HKLM\Software\Microsoft\Office\12.0\Access Connectivity Engine\Engines
    SandboxMode (DWORD Value)

...to either 0 or 2:

SETTING DESCRIPTION
   0    Sandbox mode is disabled at all times.
   1    Sandbox mode is used for Access, but not for non-Access programs.
   2    Sandbox mode is used for non-Access programs, but not for Access.
   3    Sandbox mode is used at all times. This is the default value.

Can't push to GitHub because of large file which I already deleted

I had a similar issue and used the step above to remove the file. It worked perfectly.

I then got an error on a second file that I needed to remove: remote: error: File <path/filename> is 109.99 MB; this exceeds GitHub's file size limit of 100.00 MB

I tried the same step, got an error: "A previous backup already exists in <path/filename>"

From research on this website I used the command: git filter-branch --force --index-filter "git rm --cached --ignore-unmatch <path/filename>" --prune-empty --tag-name-filter cat -- --all

Worked great, and the large files were removed.

Unbelievably, the push still failed with another error: error: RPC failed; curl 56 OpenSSL SSL_read: SSL_ERROR_SYSCALL, errno 104 fatal: The remote end hung up unexpectedly

This I fixed by directly modifying the .git config file - postBuffer = 999999999

After that the push went through!

How to pass boolean parameter value in pipeline to downstream jobs?

Assuming

value: update_composer

was the issue, try

value: Boolean.valueOf(update_composer)

is there a less cumbersome way in which I can just pass ALL the pipeline parameters to the downstream job

Not that I know of, at least not without using Jenkins API calls and disabling the Groovy sandbox.

Find ALL tweets from a user (not just the first 3,200)

Not all twitter API users are created equal - some are more equal than others.

https://dev.twitter.com/docs/streaming-api/methods

For thine not that equal they suggest creative using of other techniques. You may get more luck by using search api calls with time / id limitation

css overflow - only 1 line of text

If you want to restrict it to one line, use white-space: nowrap; on the div.

Most efficient way to increment a Map value in Java

As a follow-up to my own comment: Trove looks like the way to go. If, for whatever reason, you wanted to stick with the standard JDK, ConcurrentMap and AtomicLong can make the code a tiny bit nicer, though YMMV.

    final ConcurrentMap<String, AtomicLong> map = new ConcurrentHashMap<String, AtomicLong>();
    map.putIfAbsent("foo", new AtomicLong(0));
    map.get("foo").incrementAndGet();

will leave 1 as the value in the map for foo. Realistically, increased friendliness to threading is all that this approach has to recommend it.

How do I show the number keyboard on an EditText in android?

editText.setRawInputType(InputType.TYPE_CLASS_NUMBER);

Iterating Over Dictionary Key Values Corresponding to List in Python

You have several options for iterating over a dictionary.

If you iterate over the dictionary itself (for team in league), you will be iterating over the keys of the dictionary. When looping with a for loop, the behavior will be the same whether you loop over the dict (league) itself, or league.keys():

for team in league.keys():
    runs_scored, runs_allowed = map(float, league[team])

You can also iterate over both the keys and the values at once by iterating over league.items():

for team, runs in league.items():
    runs_scored, runs_allowed = map(float, runs)

You can even perform your tuple unpacking while iterating:

for team, (runs_scored, runs_allowed) in league.items():
    runs_scored = float(runs_scored)
    runs_allowed = float(runs_allowed)

Mvn install or Mvn package

from http://maven.apache.org/guides/getting-started/maven-in-five-minutes.html

package: take the compiled code and package it in its distributable format, such as a JAR.

install: install the package into the local repository, for use as a dependency in other projects locally

So the answer to your question is, it depends on whether you want it in installed into your local repo. Install will also run package because it's higher up in the goal phase stack.

jQuery serialize does not register checkboxes

Try this:

$(':input[type="checkbox"]:checked').map(function(){return this.value}).get();

What does ^M character mean in Vim?

I removed them all with sed:

sed -i -e 's/\r//g' <filename>

Could also replace with a different string or character. If there aren't line breaks already for example you can turn \r into \n:

sed -i -e 's/\r/\n/g' <filename>

Those sed commands work on the GNU/Linux version of sed but may need tweaking on BSDs (including macOS).

How to use opencv in using Gradle?

If you don't want to use JavaCV this works for me...

Step 1- Download the Resources

Download OpenCV Android SDK from http://opencv.org/downloads.html

Step 2 - Copying the OpenCV binaries into your APK

Copy libopencv_info.so & libopencv_java.so from

OpenCV-2.?.?-android-sdk -> sdk -> native -> libs -> armeabi-v7a

to

Project Root -> Your Project -> lib - > armeabi-v7a

Zip the lib folder up and rename that zip to whatever-v7a.jar.

Copy this .jar file and place it in here in your project

Project Root -> Your Project -> libs

Add this line to your projects build.gradle in the dependencies section

compile files('libs/whatever-v7a.jar')

When you compile now you will probably see your .apk is about 4mb bigger.

(Repeat for "armeabi" if you want to support ARMv6 too, likely not needed anymore.)

Step 3 - Adding the java sdk to your project

Copy the java folder from here

OpenCV-2.?.?-android-sdk -> sdk

to

Project Root -> Your Project -> libs (Same place as your .jar file);

(You can rename the 'java' folder name to 'OpenCV')

In this freshly copied folder add a typical build.gradle file; I used this:

buildscript {
    repositories {
        mavenCentral()
    }
    dependencies {
        classpath 'com.android.tools.build:gradle:0.6.+'
    }
}

apply plugin: 'android-library'

repositories {
    mavenCentral();
}

android {
    compileSdkVersion 19
    buildToolsVersion "19"

    defaultConfig {
        minSdkVersion 15
        targetSdkVersion 19
    }

    sourceSets {
        main {
            manifest.srcFile 'AndroidManifest.xml'
            java.srcDirs = ['src']
            resources.srcDirs = ['src']
            aidl.srcDirs = ['src']
            renderscript.srcDirs = ['src']
            res.srcDirs = ['res']
            assets.srcDirs = ['assets']
        }
    }
}

In your Project Root settings.gradle file change it too look something like this:

include ':Project Name:libs:OpenCV', ':Project Name'

In your Project Root -> Project Name -> build.gradle file in the dependencies section add this line:

compile project(':Project Name:libs:OpenCV')

Step 4 - Using OpenCV in your project

Rebuild and you should be able to import and start using OpenCV in your project.

import org.opencv.android.OpenCVLoader;
...
if (!OpenCVLoader.initDebug()) {}

I know this if a bit of hack but I figured I would post it anyway.

How can I use std::maps with user-defined types as key?

By default std::map (and std::set) use operator< to determine sorting. Therefore, you need to define operator< on your class.

Two objects are deemed equivalent if !(a < b) && !(b < a).

If, for some reason, you'd like to use a different comparator, the third template argument of the map can be changed, to std::greater, for example.

Adding elements to object

if you not design to do loop with in JS e.g. pass to PHP to do loop for you

let decision = {}
decision[code+'#'+row] = event.target.value

this concept may help a bit

What is the documents directory (NSDocumentDirectory)?

Your app only (on a non-jailbroken device) runs in a "sandboxed" environment. This means that it can only access files and directories within its own contents. For example Documents and Library.

See the iOS Application Programming Guide.

To access the Documents directory of your applications sandbox, you can use the following:

iOS 8 and newer, this is the recommended method

+ (NSURL *)applicationDocumentsDirectory
{
     return [[[NSFileManager defaultManager] URLsForDirectory:NSDocumentDirectory inDomains:NSUserDomainMask] lastObject];
}

if you need to support iOS 7 or earlier

+ (NSString *) applicationDocumentsDirectory 
{    
    NSArray *paths = NSSearchPathForDirectoriesInDomains(NSDocumentDirectory, NSUserDomainMask, YES);
    NSString *basePath = paths.firstObject;
    return basePath;
}

This Documents directory allows you to store files and subdirectories your app creates or may need.

To access files in the Library directory of your apps sandbox use (in place of paths above):

[NSSearchPathForDirectoriesInDomains(NSLibraryDirectory, NSUserDomainMask, YES) objectAtIndex:0]

How to get an Instagram Access Token

If you don't want to build your server side, like only developing on a client side (web app or a mobile app) , you could choose an Implicit Authentication .

As the document saying , first make a https request with

https://api.instagram.com/oauth/authorize/?client_id=CLIENT-ID&redirect_uri=REDIRECT-URI&response_type=token

Fill in your CLIENT-ID and REDIRECT-URL you designated.

Then that's going to the log in page , but the most important thing is how to get the access token after the user correctly logging in.

After the user click the log in button with both correct account and password, the web page will redirect to the url you designated followed by a new access token.

http://your-redirect-uri#access_token=ACCESS-TOKEN

I'm not familiar with javascript , but in Android studio , that's an easy way to add a listener which listen to the event the web page override the url to the new url (redirect event) , then it will pass the redirect url string to you , so you can easily split it to get the access-token like:

String access_token = url.split("=")[1];

Means to break the url into the string array in each "=" character , then the access token obviously exists at [1].

Namenode not getting started

If anyone using hadoop1.2.1 version and not able to run namenode, go to core-site.xml, and change dfs.default.name to fs.default.name.

And then format the namenode using $hadoop namenode -format.

Finally run the hdfs using start-dfs.sh and check for service using jps..

AJAX post error : Refused to set unsafe header "Connection"

Remove these two lines:

xmlHttp.setRequestHeader("Content-length", params.length);
xmlHttp.setRequestHeader("Connection", "close");

XMLHttpRequest isn't allowed to set these headers, they are being set automatically by the browser. The reason is that by manipulating these headers you might be able to trick the server into accepting a second request through the same connection, one that wouldn't go through the usual security checks - that would be a security vulnerability in the browser.

Type of expression is ambiguous without more context Swift

Explicitly declaring the inputs for that mapping function should do the trick:

let imageToDeleteParameters  = imagesToDelete.map {
    (whatever : WhateverClass) -> Dictionary<String, Any> in
    ["id": whatever.id, "url": whatever.url.absoluteString, "_destroy": true]
}

Substitute the real class of "$0" for "WhateverClass" in that code snippet, and it should work.

How to pass parameter to function using in addEventListener?

When you use addEventListener, this will be bound automatically. So if you want a reference to the element on which the event handler is installed, just use this from within your function:

productLineSelect.addEventListener('change',getSelection,false);

function getSelection(){
    var value = sel.options[this.selectedIndex].value;
    alert(value);
}

If you want to pass in some other argument from the context where you call addEventListener, you can use a closure, like this:

productLineSelect.addEventListener('change', function(){ 
    // pass in `this` (the element), and someOtherVar
    getSelection(this, someOtherVar); 
},false);

function getSelection(sel, someOtherVar){
    var value = sel.options[sel.selectedIndex].value;
    alert(value);
    alert(someOtherVar);
}

How can I check the current status of the GPS receiver?

I may be wrong but it seems people seem to be going way off-topic for

i just need to know if the gps icon at the top of the screen is blinking (no actual fix)

That is easily done with

LocationManager lm = (LocationManager) getSystemService(LOCATION_SERVICE);
boolean gps_on = lm.isProviderEnabled(LocationManager.GPS_PROVIDER);

To see if you have a solid fix, things get a little trickier:

public class whatever extends Activity {
    LocationManager lm;
    Location loc;
    public void onCreate(Bundle savedInstanceState) {
        super.onCreate(savedInstanceState);
        setContentView(R.layout.main);        
        lm = (LocationManager) getSystemService(LOCATION_SERVICE);
        loc = null;
        request_updates();        
    }

    private void request_updates() {
        if (lm.isProviderEnabled(LocationManager.GPS_PROVIDER)) {
            // GPS is enabled on device so lets add a loopback for this locationmanager
            lm.requestLocationUpdates(LocationManager.GPS_PROVIDER,0, 0, locationListener);
        }      
    }

    LocationListener locationListener = new LocationListener() {
        public void onLocationChanged(Location location) {
            // Each time the location is changed we assign loc
            loc = location;
        }

         // Need these even if they do nothing. Can't remember why.
         public void onProviderDisabled(String arg0) {}
         public void onProviderEnabled(String provider) {}
         public void onStatusChanged(String provider, int status, Bundle extras) {}
    };

Now whenever you want to see if you have fix?

if (loc != null){
    // Our location has changed at least once
    blah.....
}

If you want to be fancy you can always have a timeout using System.currentTimeMillis() and loc.getTime()

Works reliably, at least on an N1 since 2.1.

MySQL remove all whitespaces from the entire column

Since the question is how to replace ALL whitespaces

UPDATE `table` 
SET `col_name` = REPLACE
(REPLACE(REPLACE(`col_name`, ' ', ''), '\t', ''), '\n', '');

Returning an empty array

Both foo() and bar() may generate warnings in some IDEs. For example, IntelliJ IDEA will generate a Allocation of zero-length array warning.

An alternative approach is to use Apache Commons Lang 3 ArrayUtils.toArray() function with empty arguments:

public File[] bazz() {
    return ArrayUtils.toArray();
}

This approach is both performance and IDE friendly, yet requires a 3rd party dependency. However, if you already have commons-lang3 in your classpath, you could even use statically-defined empty arrays for primitive types:

public String[] bazz() {
    return ArrayUtils.EMPTY_STRING_ARRAY;
}

Unknown version of Tomcat was specified in Eclipse

I am on MacOS and installed tomcat using homebrew, Following path fixed my problem

/usr/local/Cellar/tomcat/9.0.14/libexec

Swift: How to get substring from start to last index of character

The one thing that adds clatter is the repeated stringVar:

stringVar[stringVar.index(stringVar.startIndex, offsetBy: ...)

In Swift 4

An extension can reduce some of that:

extension String {

    func index(at location: Int) -> String.Index {
        return self.index(self.startIndex, offsetBy: location)
    }
}

Then, usage:

let string = "abcde"

let to = string[..<string.index(at: 3)] // abc
let from = string[string.index(at: 3)...] // de

It should be noted that to and from are type Substring (or String.SubSequance). They do not allocate new strings and are more efficient for processing.

To get back a String type, Substring needs to be casted back to String:

let backToString = String(from)

This is where a string is finally allocated.

How to restart kubernetes nodes?

I had this problem too but it looks like it depends on the Kubernetes offering and how everything was installed. In Azure, if you are using acs-engine install, you can find the shell script that is actually being run to provision it at:

/opt/azure/containers/provision.sh

To get a more fine-grained understanding, just read through it and run the commands that it specifies. For me, I had to run as root:

systemctl enable kubectl
systemctl restart kubectl

I don't know if the enable is necessary and I can't say if these will work with your particular installation, but it definitely worked for me.

How to read a CSV file into a .NET Datatable

    private static DataTable LoadCsvData(string refPath)
    {
        var cfg = new Configuration() { Delimiter = ",", HasHeaderRecord = true };
        var result = new DataTable();
        using (var sr = new StreamReader(refPath, Encoding.UTF8, false, 16384 * 2))
        {
            using (var rdr = new CsvReader(sr, cfg))
            using (var dataRdr = new CsvDataReader(rdr))
            {
                result.Load(dataRdr);
            }
        }
        return result;
    }

using: https://joshclose.github.io/CsvHelper/

complex if statement in python

I think the most pythonic way to do this for me, will be

elif var in [80,443] + range(1024,65535):

although it could take a little time and memory (it's generating numbers from 1024 to 65535). If there's a problem with that, I'll do:

elif 1024 <= var <= 65535 or var in [80,443]:

"An attempt was made to access a socket in a way forbidden by its access permissions" while using SMTP

Ok, so very important to realize the implications here.

Docs say that SSL over 465 is NOT supported in SmtpClient.

Seems like you have no choice but to use STARTTLS which may not be supported by your mail host. You may have to use a different library if your host requires use of SSL over 465.

Quoted from http://msdn.microsoft.com/en-us/library/system.net.mail.smtpclient.enablessl(v=vs.110).aspx

The SmtpClient class only supports the SMTP Service Extension for Secure SMTP over Transport Layer Security as defined in RFC 3207. In this mode, the SMTP session begins on an unencrypted channel, then a STARTTLS command is issued by the client to the server to switch to secure communication using SSL. See RFC 3207 published by the Internet Engineering Task Force (IETF) for more information.

An alternate connection method is where an SSL session is established up front before any protocol commands are sent. This connection method is sometimes called SMTP/SSL, SMTP over SSL, or SMTPS and by default uses port 465. This alternate connection method using SSL is not currently supported.

JPQL IN clause: Java-Arrays (or Lists, Sets...)?

I had a problem with this kind of sql, I was giving empty list in IN clause(always check the list if it is not empty). Maybe my practice will help somebody.

How to convert hex to ASCII characters in the Linux shell?

Similar to my answer here: Linux shell scripting: hex number to binary string

You can do it with the same tool like this (using ascii printable character instead of 5a):

echo -n 616263 | cryptocli dd -decoders hex

Will produce the following result:

abcd

Extract specific columns from delimited file using Awk

Not using awk but the simplest way I was able to get this done was to just use csvtool. I had other use cases as well to use csvtool and it can handle the quotes or delimiters appropriately if they appear within the column data itself.

csvtool format '%(2)\n' input.csv
csvtool format '%(2),%(3),%(4)\n' input.csv

Replacing 2 with the column number will effectively extract the column data you are looking for.

Text size and different android screen sizes

Everyone can use the below mentioned android library that is the easiest way to make text sizes compatible with almost all devices screens. It actually developed on the basis of new android configuration qualifiers for screen size (introduced in Android 3.2) SmallestWidth swdp.

https://github.com/intuit/sdp

Java Equivalent of C# async/await?

There isn't anything native to java that lets you do this like async/await keywords, but what you can do if you really want to is use a CountDownLatch. You could then imitate async/await by passing this around (at least in Java7). This is a common practice in Android unit testing where we have to make an async call (usually a runnable posted by a handler), and then await for the result (count down).

Using this however inside your application as opposed to your test is NOT what I am recommending. That would be extremely shoddy as CountDownLatch depends on you effectively counting down the right number of times and in the right places.

How do I append a node to an existing XML file in java

The following complete example will read an existing server.xml file from the current directory, append a new Server and re-write the file to server.xml. It does not work without an existing .xml file, so you will need to modify the code to handle that case.

import java.util.*;
import javax.xml.transform.*;
import javax.xml.transform.stream.*;
import javax.xml.transform.dom.*;
import org.w3c.dom.*;
import javax.xml.parsers.*;

public class AddXmlNode {
    public static void main(String[] args) throws Exception {

        DocumentBuilderFactory documentBuilderFactory = DocumentBuilderFactory.newInstance();
        DocumentBuilder documentBuilder = documentBuilderFactory.newDocumentBuilder();
        Document document = documentBuilder.parse("server.xml");
        Element root = document.getDocumentElement();

        Collection<Server> servers = new ArrayList<Server>();
        servers.add(new Server());

        for (Server server : servers) {
            // server elements
            Element newServer = document.createElement("server");

            Element name = document.createElement("name");
            name.appendChild(document.createTextNode(server.getName()));
            newServer.appendChild(name);

            Element port = document.createElement("port");
            port.appendChild(document.createTextNode(Integer.toString(server.getPort())));
            newServer.appendChild(port);

            root.appendChild(newServer);
        }

        DOMSource source = new DOMSource(document);

        TransformerFactory transformerFactory = TransformerFactory.newInstance();
        Transformer transformer = transformerFactory.newTransformer();
        StreamResult result = new StreamResult("server.xml");
        transformer.transform(source, result);
    }

    public static class Server {
        public String getName() { return "foo"; }
        public Integer getPort() { return 12345; }
    }
}

Example server.xml file:

<?xml version="1.0" encoding="UTF-8" standalone="yes"?>
<Servers>
  <server>
    <name>something</name>
    <port>port</port>
  </server>
</Servers>

The main change to your code is not creating a new "root" element. The above example just uses the current root node from the existing server.xml and then just appends a new Server element and re-writes the file.

How to see which flags -march=native will activate?

I'm going to throw my two cents into this question and suggest a slightly more verbose extension of elias's answer. As of gcc 4.6, running of gcc -march=native -v -E - < /dev/null emits an increasing amount of spam in the form of superfluous -mno-* flags. The following will strip these:

gcc -march=native -v -E - < /dev/null 2>&1 | grep cc1 | perl -pe 's/ -mno-\S+//g; s/^.* - //g;'

However, I have only verified the correctness of this on two different CPUs (an Intel Core2 and AMD Phenom), so I suggest also running the following script to be sure that all of these -mno-* flags can be safely stripped.

2021 EDIT: There are indeed machines where -march=native uses a particular -march value, but must disable some implied ISAs (Instruction Set Architecture) with -mno-*.

#!/bin/bash

gcc_cmd="gcc"

# Optionally supply path to gcc as first argument
if (($#)); then
    gcc_cmd="$1"
fi

with_mno=$(
    "${gcc_cmd}" -march=native -mtune=native -v -E - < /dev/null 2>&1 |
    grep cc1 |
    perl -pe 's/^.* - //g;'
)
without_mno=$(echo "${with_mno}" | perl -pe 's/ -mno-\S+//g;')

"${gcc_cmd}" ${with_mno}    -dM -E - < /dev/null > /tmp/gcctest.a.$$
"${gcc_cmd}" ${without_mno} -dM -E - < /dev/null > /tmp/gcctest.b.$$

if diff -u /tmp/gcctest.{a,b}.$$; then
    echo "Safe to strip -mno-* options."
else
    echo
    echo "WARNING! Some -mno-* options are needed!"
    exit 1
fi

rm /tmp/gcctest.{a,b}.$$

I haven't found a difference between gcc -march=native -v -E - < /dev/null and gcc -march=native -### -E - < /dev/null other than some parameters being quoted -- and parameters that contain no special characters, so I'm not sure under what circumstances this makes any real difference.

Finally, note that --march=native was introduced in gcc 4.2, prior to which it is just an unrecognized argument.

Storing Images in DB - Yea or Nay?

Im my experience I had to manage both situations: images stored in database and images on the file system with path stored in db.

The first solution, images in database, is somewhat "cleaner" as your data access layer will have to deal only with database objects; but this is good only when you have to deal with low numbers.

Obviously database access performance when you deal with binary large objects is degrading, and the database dimensions will grow a lot, causing again performance loss... and normally database space is much more expensive than file system space.

On the other hand having large binary objects stored in file system will cause you to have backup plans that have to consider both database and file system, and this can be an issue for some systems.

Another reason to go for file system is when you have to share your images data (or sounds, video, whatever) with third party access: in this days I'm developing a web app that uses images that have to be accessed from "outside" my web farm in such a way that a database access to retrieve binary data is simply impossible. So sometimes there are also design considerations that will drive you to a choice.

Consider also, when making this choice, if you have to deal with permission and authentication when accessing binary objects: these requisites normally can be solved in an easier way when data are stored in db.

Generate random numbers uniformly over an entire range

If RAND_MAX is 32767, you can double the number of bits easily.

int BigRand()
{
    assert(INT_MAX/(RAND_MAX+1) > RAND_MAX);
    return rand() * (RAND_MAX+1) + rand();
}

Passing structs to functions

Passing structs to functions by reference: simply :)

#define maxn 1000

struct solotion
{
    int sol[maxn];
    int arry_h[maxn];
    int cat[maxn];
    int scor[maxn];

};

void inser(solotion &come){
    come.sol[0]=2;
}

void initial(solotion &come){
    for(int i=0;i<maxn;i++)
        come.sol[i]=0;
}

int main()
{
    solotion sol1;
    inser(sol1);
    solotion sol2;
    initial(sol2);
}

How do I add a delay in a JavaScript loop?

var count = 0;

//Parameters:
//  array: []
//  fnc: function (the business logic in form of function-,what you want to execute)
//  delay: milisecond  

function delayLoop(array,fnc,delay){
    if(!array || array.legth == 0)return false;
    setTimeout(function(data){ 
        var data = array[count++];
        fnc && fnc(data);
        //recursion...
        if(count < array.length)
            delayLoop(array,fnc,delay);
        else count = 0;     
    },delay);
}

Session unset, or session_destroy?

Something to be aware of, the $_SESSION variables are still set in the same page after calling session_destroy() where as this is not the case when using unset($_SESSION) or $_SESSION = array(). Also, unset($_SESSION) blows away the $_SESSION superglobal so only do this when you're destroying a session.

With all that said, it's best to do like the PHP docs has it in the first example for session_destroy().

Execute PHP scripts within Node.js web server

Take a look here: https://github.com/davidcoallier/node-php

From their read me:

Inline PHP Server Running on Node.js

Be worried, be very worried. The name NodePHP takes its name from the fact that we are effectively turning a nice Node.js server into a FastCGI interface that interacts with PHP-FPM.

This is omega-alpha-super-beta-proof-of-concept but it already runs a few simple scripts. Mostly done for my talks on Node.js for PHP Developers this turns out to be quite an interesting project that we are most likely be going to use with Orchestra when we decide to release our Inline PHP server that allows people to run PHP without Apache, Nginx or any webserver.

Yes this goes against all ideas and concepts of Node.js but the idea is to be able to create a web-server directly from any working directory to allow developers to get going even faster than it was before. No need to create vhosts or server blocks ore modify your /etc/hosts anymore.

How do I replace multiple spaces with a single space in C#?

no Regex, no Linq... removes leading and trailing spaces as well as reducing any embedded multiple space segments to one space

string myString = "   0 1 2  3   4               5  ";
myString = string.Join(" ", myString.Split(new char[] { ' ' }, 
StringSplitOptions.RemoveEmptyEntries));

result:"0 1 2 3 4 5"

#1214 - The used table type doesn't support FULLTEXT indexes

Only MyISAM allows for FULLTEXT, as seen here.

Try this:

CREATE TABLE gamemech_chat (
  id bigint(20) unsigned NOT NULL auto_increment,
  from_userid varchar(50) NOT NULL default '0',
  to_userid varchar(50) NOT NULL default '0',
  text text NOT NULL,
  systemtext text NOT NULL,
  timestamp datetime NOT NULL default '0000-00-00 00:00:00',
  chatroom bigint(20) NOT NULL default '0',
  PRIMARY KEY  (id),
  KEY from_userid (from_userid),
  FULLTEXT KEY from_userid_2 (from_userid),
  KEY chatroom (chatroom),
  KEY timestamp (timestamp)
) ENGINE=MyISAM;

Materialize CSS - Select Doesn't Seem to Render

This worked for me, no jquery or select wrapper with input class, just material.js and this vanilla js:

document.addEventListener('DOMContentLoaded', function() {
    var elems = document.querySelectorAll('select');
    var instances = M.FormSelect.init(elems);
});

enter image description here enter image description here

As you can tell I got the materialize css actual style and not the browsers default.

Horizontal swipe slider with jQuery and touch devices support?

Ive found another: http://swipejs.com/

seems to work nicely however I encounter an issue with it when paired with bootstrap on the OS X version of Chrome. If total cross-browser compatibility isn't an issue, then you're golden.

How to create a fixed-size array of objects

Fixed-length arrays are not yet supported. What does that actually mean? Not that you can't create an array of n many things — obviously you can just do let a = [ 1, 2, 3 ] to get an array of three Ints. It means simply that array size is not something that you can declare as type information.

If you want an array of nils, you'll first need an array of an optional type — [SKSpriteNode?], not [SKSpriteNode] — if you declare a variable of non-optional type, whether it's an array or a single value, it cannot be nil. (Also note that [SKSpriteNode?] is different from [SKSpriteNode]?... you want an array of optionals, not an optional array.)

Swift is very explicit by design about requiring that variables be initialized, because assumptions about the content of uninitialized references are one of the ways that programs in C (and some other languages) can become buggy. So, you need to explicitly ask for an [SKSpriteNode?] array that contains 64 nils:

var sprites = [SKSpriteNode?](repeating: nil, count: 64)

This actually returns a [SKSpriteNode?]?, though: an optional array of optional sprites. (A bit odd, since init(count:,repeatedValue:) shouldn't be able to return nil.) To work with the array, you'll need to unwrap it. There's a few ways to do that, but in this case I'd favor optional binding syntax:

if var sprites = [SKSpriteNode?](repeating: nil, count: 64){
    sprites[0] = pawnSprite
}

Can CSS detect the number of children an element has?

NOTE: This solution will return the children of sets of certain lengths, not the parent element as you have asked. Hopefully, it's still useful.

Andre Luis came up with a method: http://lea.verou.me/2011/01/styling-children-based-on-their-number-with-css3/ Unfortunately, it only works in IE9 and above.

Essentially, you combine :nth-child() with other pseudo classes that deal with the position of an element. This approach allows you to specify elements from sets of elements with specific lengths.

For instance :nth-child(1):nth-last-child(3) matches the first element in a set while also being the 3rd element from the end of the set. This does two things: guarantees that the set only has three elements and that we have the first of the three. To specify the second element of the three element set, we'd use :nth-child(2):nth-last-child(2).


Example 1 - Select all list elements if set has three elements:

li:nth-child(1):nth-last-child(3),
li:nth-child(2):nth-last-child(2),
li:nth-child(3):nth-last-child(1) {
    width: 33.3333%;
}

Example 1 alternative from Lea Verou:

li:first-child:nth-last-child(3),
li:first-child:nth-last-child(3) ~ li {
    width: 33.3333%;
}


Example 2 - target last element of set with three list elements:

li:nth-child(3):last-child {
    /* I'm the last of three */
}

Example 2 alternative:

li:nth-child(3):nth-last-child(1) {
    /* I'm the last of three */
}


Example 3 - target second element of set with four list elements:

li:nth-child(2):nth-last-child(3) {
    /* I'm the second of four */
}

Inserting a Python datetime.datetime object into MySQL

What database are you connecting to? I know Oracle can be picky about date formats and likes ISO 8601 format.

**Note: Oops, I just read you are on MySQL. Just format the date and try it as a separate direct SQL call to test.

In Python, you can get an ISO date like

now.isoformat()

For instance, Oracle likes dates like

insert into x values(99, '31-may-09');

Depending on your database, if it is Oracle you might need to TO_DATE it:

insert into x
values(99, to_date('2009/05/31:12:00:00AM', 'yyyy/mm/dd:hh:mi:ssam'));

The general usage of TO_DATE is:

TO_DATE(<string>, '<format>')

If using another database (I saw the cursor and thought Oracle; I could be wrong) then check their date format tools. For MySQL it is DATE_FORMAT() and SQL Server it is CONVERT.

Also using a tool like SQLAlchemy will remove differences like these and make your life easy.

How to write and read a file with a HashMap?

HashMap implements Serializable so you can use normal serialization to write hashmap to file

Here is the link for Java - Serialization example

git push rejected

Jarret Hardie is correct. Or, first merge your changes back into master and then try the push. By default, git push pushes all branches that have names that match on the remote -- and no others. So those are your two choices -- either specify it explicitly like Jarret said or merge back to a common branch and then push.

There's been talk about this on the Git mail list and it's clear that this behavior is not about to change anytime soon -- many developers rely on this behavior in their workflows.

Edit/Clarification

Assuming your upstreammaster branch is ready to push then you could do this:

  1. Pull in any changes from the upstream.

    $ git pull upstream master

  2. Switch to my local master branch

    $ git checkout master

  3. Merge changes in from upstreammaster

    $ git merge upstreammaster

  4. Push my changes up

    $ git push upstream

Another thing that you may want to do before pushing is to rebase your changes against upstream/master so that your commits are all together. You can either do that as a separate step between #1 and #2 above (git rebase upstream/master) or you can do it as part of your pull (git pull --rebase upstream master)

Why do I get the error "Unsafe code may only appear if compiling with /unsafe"?

To use unsafe code blocks, the project has to be compiled with the /unsafe switch on.

Open the properties for the project, go to the Build tab and check the Allow unsafe code checkbox.

How to calculate the inverse of the normal cumulative distribution function in python?

NORMSINV (mentioned in a comment) is the inverse of the CDF of the standard normal distribution. Using scipy, you can compute this with the ppf method of the scipy.stats.norm object. The acronym ppf stands for percent point function, which is another name for the quantile function.

In [20]: from scipy.stats import norm

In [21]: norm.ppf(0.95)
Out[21]: 1.6448536269514722

Check that it is the inverse of the CDF:

In [34]: norm.cdf(norm.ppf(0.95))
Out[34]: 0.94999999999999996

By default, norm.ppf uses mean=0 and stddev=1, which is the "standard" normal distribution. You can use a different mean and standard deviation by specifying the loc and scale arguments, respectively.

In [35]: norm.ppf(0.95, loc=10, scale=2)
Out[35]: 13.289707253902945

If you look at the source code for scipy.stats.norm, you'll find that the ppf method ultimately calls scipy.special.ndtri. So to compute the inverse of the CDF of the standard normal distribution, you could use that function directly:

In [43]: from scipy.special import ndtri

In [44]: ndtri(0.95)
Out[44]: 1.6448536269514722

Docker: How to use bash with an Alpine based docker image?

RUN /bin/sh -c "apk add --no-cache bash"

worked for me.

Visual Studio debugger error: Unable to start program Specified file cannot be found

I think that what you have to check is:

  1. if the target EXE is correctly configured in the project settings ("command", in the debugging tab). Since all individual projects run when you start debugging it's well possible that only the debugging target for the "ALL" solution is missing, check which project is currently active (you can also select the debugger target by changing the active project).

  2. dependencies (DLLs) are also located at the target debugee directory or can be loaded (you can use the "depends.exe" tool for checking dependencies of an executable or DLL).

Failed to execute goal org.codehaus.mojo:exec-maven-plugin:1.2:java (default-cli)

Your problem is that you have declare twice the exec-maven-plugin :

<plugin>
    <groupId>org.codehaus.mojo</groupId>
    <artifactId>exec-maven-plugin</artifactId>
    <version>1.2.1</version>
    <executions>
        <execution>
            <goals>
                <goal>java</goal>
            </goals>
        </execution>
    </executions>
    <configuration>
        <mainClass>C:\apache-camel-2.11.0\examples\camel-example-smooks-  
      integration\src\main\java\example\Main< /mainClass>
    </configuration>
</plugin>

...

< plugin>
    < groupId>org.codehaus.mojo</groupId>
    < artifactId>exec-maven-plugin</artifactId>
    < version>1.2</version>
< /plugin>

Plot yerr/xerr as shaded region rather than error bars

Ignoring the smooth interpolation between points in your example graph (that would require doing some manual interpolation, or just have a higher resolution of your data), you can use pyplot.fill_between():

from matplotlib import pyplot as plt
import numpy as np

x = np.linspace(0, 30, 30)
y = np.sin(x/6*np.pi)
error = np.random.normal(0.1, 0.02, size=y.shape)
y += np.random.normal(0, 0.1, size=y.shape)

plt.plot(x, y, 'k-')
plt.fill_between(x, y-error, y+error)
plt.show()

enter image description here

See also the matplotlib examples.

How to cut an entire line in vim and paste it?

  1. Go to the line, and first press esc, and then Shift + v.

(This would have highlighted the line)

  1. press d

(The line is now deleted)

  1. Go to the location, where you wanted to paste the line, and hit p.

In a nutshell,

Esc -> Shift + v -> d -> p

Replacing few values in a pandas dataframe column with another value

This solution will change the existing dataframe itself:

mydf = pd.DataFrame({"BrandName":["A", "B", "ABC", "D", "AB"], "Speciality":["H", "I", "J", "K", "L"]})
mydf["BrandName"].replace(["ABC", "AB"], "A", inplace=True)

How to set page content to the middle of screen?

HTML

<!DOCTYPE html>
<html>
    <head>
        <title>Center</title>        
    </head>
    <body>
        <div id="main_body">
          some text
        </div>
    </body>
</html>

CSS

body
{
   width: 100%;
   Height: 100%;
}
#main_body
{
    background: #ff3333;
    width: 200px;
    position: absolute;
}?

JS ( jQuery )

$(function(){
    var windowHeight = $(window).height();
    var windowWidth = $(window).width();
    var main = $("#main_body");    
    $("#main_body").css({ top: ((windowHeight / 2) - (main.height() / 2)) + "px",
                          left:((windowWidth / 2) - (main.width() / 2)) + "px" });
});

See example here

How to determine if a list of polygon points are in clockwise order?

Find the vertex with smallest y (and largest x if there are ties). Let the vertex be A and the previous vertex in the list be B and the next vertex in the list be C. Now compute the sign of the cross product of AB and AC.


References:

Positioning background image, adding padding

Updated Answer:

It's been commented multiple times that this is not the correct answer to this question, and I agree. Back when this answer was written, IE 9 was still new (about 8 months old) and many developers including myself needed a solution for <= IE 9. IE 9 is when IE started supporting background-origin. However, it's been over six and a half years, so here's the updated solution which I highly recommend over using an actual border. In case < IE 9 support is needed. My original answer can be found below the demo snippet. It uses an opaque border to simulate padding for background images.

_x000D_
_x000D_
#hello {
  padding-right: 10px;
  background-color:green;
  background: url("https://placehold.it/15/5C5/FFF") no-repeat scroll right center #e8e8e8;
  background-origin: content-box;
}
_x000D_
<p id="hello">I want the background icon to have padding to it too!I want the background icon twant the background icon to have padding to it too!I want the background icon to have padding to it too!I want the background icon to have padding to it too!</p>
_x000D_
_x000D_
_x000D_

Original Answer:

you can fake it with a 10px border of the same color as the background:

http://jsbin.com/eparad/edit#javascript,html,live

#hello {
   border: 10px solid #e8e8e8;
   background-color: green;
   background: url("http://www.costascuisine.com/images/buttons/collapseIcon.gif")
               no-repeat scroll right center #e8e8e8;
}

Best way to restrict a text field to numbers only?

shorter way and easy to understand:

$('#someID').keypress(function(e) { 
    var k = e.which;
    if (k <= 48 || k >= 58) {e.preventDefault()};
});

Fetch the row which has the Max value for a column

Use the code:

select T.UserId,T.dt from (select UserId,max(dt) 
over (partition by UserId) as dt from t_users)T where T.dt=dt;

This will retrieve the results, irrespective of duplicate values for UserId. If your UserId is unique, well it becomes more simple:

select UserId,max(dt) from t_users group by UserId;

Spark : how to run spark file from spark shell

You can run as you run your shell script. This example to run from command line environment example

./bin/spark-shell :- this is the path of your spark-shell under bin /home/fold1/spark_program.py :- This is the path where your python program is there.

So:

./bin.spark-shell /home/fold1/spark_prohram.py

How does Spring autowire by name when more than one matching bean is found?

in some case you can use annotation @Primary.

@Primary
class USA implements Country {}

This way it will be selected as the default autowire candididate, with no need to autowire-candidate on the other bean.

for mo deatils look at Autowiring two beans implementing same interface - how to set default bean to autowire?

"Keep Me Logged In" - the best approach

My solution is like this. It's not 100% bulletproof but I think it will save you for the most of the cases.

When user logged in successfully create a string with this information:

$data = (SALT + ":" + hash(User Agent) + ":" + username 
                     + ":" + LoginTimestamp + ":"+ SALT)

Encrypt $data, set type to HttpOnly and set cookie.

When user come back to your site, Make this steps:

  1. Get cookie data. Remove dangerous characters inside cookie. Explode it with : character.
  2. Check validity. If cookie is older than X days then redirect user to login page.
  3. If cookie is not old; Get latest password change time from database. If password is changed after user's last login redirect user to login page.
  4. If pass wasn't changed recently; Get user's current browser agent. Check whether (currentUserAgentHash == cookieUserAgentHash). IF agents are same go to next step, else redirect to login page.
  5. If all steps passed successfully authorize username.

If user signouts, remove this cookie. Create new cookie if user re-logins.

JavaScript Regular Expression Email Validation

I've been using this function for a while. it returns a boolean value.

// Validates email address of course.
function validEmail(e) {
    var filter = /^\s*[\w\-\+_]+(\.[\w\-\+_]+)*\@[\w\-\+_]+\.[\w\-\+_]+(\.[\w\-\+_]+)*\s*$/;
    return String(e).search (filter) != -1;
}

Submit form using a button outside the <form> tag

A solution that works great for me, is still missing here. It requires having a visually hidden <submit> or <input type="submit"> element whithin the <form>, and an associated <label> element outside of it. It would look like this:

<form method="get" action="something.php">
     <input type="text" name="name" />
     <input type="submit" id="submit-form" class="hidden" />
</form>

<label for="submit-form" tabindex="0">Submit</label>

Now this link enables you to 'click' the form <submit> element by clicking the <label> element.

Using a BOOL property

There's no benefit to using properties with primitive types. @property is used with heap allocated NSObjects like NSString*, NSNumber*, UIButton*, and etc, because memory managed accessors are created for free. When you create a BOOL, the value is always allocated on the stack and does not require any special accessors to prevent memory leakage. isWorking is simply the popular way of expressing the state of a boolean value.

In another OO language you would make a variable private bool working; and two accessors: SetWorking for the setter and IsWorking for the accessor.

java.util.Date format conversion yyyy-mm-dd to mm-dd-yyyy

tl;dr

LocalDate.parse( 
    "01-23-2017" , 
    DateTimeFormatter.ofPattern( "MM-dd-uuuu" )
)

Details

I have a java.util.Date in the format yyyy-mm-dd

As other mentioned, the Date class has no format. It has a count of milliseconds since the start of 1970 in UTC. No strings attached.

java.time

The other Answers use troublesome old legacy date-time classes, now supplanted by the java.time classes.

If you have a java.util.Date, convert to a Instant object. The Instant class represents a moment on the timeline in UTC with a resolution of nanoseconds (up to nine (9) digits of a decimal fraction).

Instant instant = myUtilDate.toInstant();

Time zone

The other Answers ignore the crucial issue of time zone. Determining a date requires a time zone. For any given moment, the date varies around the globe by zone. A few minutes after midnight in Paris France is a new day, while still “yesterday” in Montréal Québec.

Define the time zone by which you want context for your Instant.

ZoneId z = ZoneId.of( "America/Montreal" );

Apply the ZoneId to get a ZonedDateTime.

ZonedDateTime zdt = instant.atZone( z );

LocalDate

If you only care about the date without a time-of-day, extract a LocalDate.

LocalDate localDate = zdt.toLocalDate();

To generate a string in standard ISO 8601 format, YYYY-MM-DD, simply call toString. The java.time classes use the standard formats by default when generating/parsing strings.

String output = localDate.toString();

2017-01-23

If you want a MM-DD-YYYY format, define a formatting pattern.

DateTimeFormatter f = DateTimeFormatter.ofPattern( "MM-dd-uuuu" );
String output = localDate.format( f );

Note that the formatting pattern codes are case-sensitive. The code in the Question incorrectly used mm (minute of hour) rather than MM (month of year).

Use the same DateTimeFormatter object for parsing. The java.time classes are thread-safe, so you can keep this object around and reuse it repeatedly even across threads.

LocalDate localDate = LocalDate.parse( "01-23-2017" , f );

About java.time

The java.time framework is built into Java 8 and later. These classes supplant the troublesome old legacy date-time classes such as java.util.Date, Calendar, & SimpleDateFormat.

The Joda-Time project, now in maintenance mode, advises migration to the java.time classes.

To learn more, see the Oracle Tutorial. And search Stack Overflow for many examples and explanations. Specification is JSR 310.

Where to obtain the java.time classes?

The ThreeTen-Extra project extends java.time with additional classes. This project is a proving ground for possible future additions to java.time. You may find some useful classes here such as Interval, YearWeek, YearQuarter, and more.

MVC4 HTTP Error 403.14 - Forbidden

Perhaps... If you happen to use the Publish Wizard (like I did) and select the "Precompile during publishing" checkbox (like I did) and see the same symptoms...

Yeah, I beat myself over the head, but after unchecking this box, a seemingly unrelated setting, all the symptoms described go away after redeploying.

Hopefully this fixes some folks.

The matching wildcard is strict, but no declaration can be found for element 'tx:annotation-driven'

You have some errors in your appcontext.xml:

  • Use *-2.5.xsd

    xsi:schemaLocation="http://www.springframework.org/schema/beans 
      http://www.springframework.org/schema/beans/spring-beans-2.5.xsd
      http://www.springframework.org/schema/aop 
      http://www.springframework.org/schema/aop/spring-aop-2.5.xsd
      http://www.springframework.org/schema/context 
      http://www.springframework.org/schema/context/spring-context-2.5.xsd
      http://www.springframework.org/schema/tx 
      http://www.springframework.org/schema/tx/spring-tx-2.5.xsd"
    
  • Typos in tx:annotation-driven and context:component-scan (. instead of -)

    <tx:annotation-driven transaction-manager="transactionManager" />
    <context:component-scan base-package="com.mmycompany" />
    

adding a datatable in a dataset

Just give any name to the DataTable Like:

DataTable dt = new DataTable();
dt = SecondDataTable.Copy();    
dt .TableName = "New Name";
DataSet.Tables.Add(dt );

How does inline Javascript (in HTML) work?

using javascript:

here input element is used

<input type="text" id="fname" onkeyup="javascript:console.log(window.event.key)">

if you want to use multiline code use curly braces after javascript:

<input type="text" id="fname" onkeyup="javascript:{ console.log(window.event.key); alert('hello'); }">

How can I align the columns of tables in Bash?

Below code has been tested and does exactly what is requested in the original question.

Parameters: %30s Column of 30 char and text right align. %10d integer notation, %10s will also work. Added clarification included on code comments.

stringarray[0]="a very long string.........."
# 28Char (max length for this column)
numberarray[0]=1122324333
# 10digits (max length for this column)
anotherfield[0]="anotherfield"
# 12Char (max length for this column)
stringarray[1]="a smaller string....."
numberarray[1]=123124343
anotherfield[1]="anotherfield"

printf "%30s %10d %13s" "${stringarray[0]}" ${numberarray[0]} "${anotherfield[0]}"
printf "\n"
printf "%30s %10d %13s" "${stringarray[1]}" ${numberarray[1]} "${anotherfield[1]}"
# a var string with spaces has to be quoted
printf "\n Next line will fail \n"      
printf "%30s %10d %13s" ${stringarray[0]} ${numberarray[0]} "${anotherfield[0]}"



  a very long string.......... 1122324333  anotherfield
         a smaller string.....  123124343  anotherfield

How do I get the day month and year from a Windows cmd.exe script?

You can use simple variable syntax, here is an example:

@echo off
set month=%date:~0,2%
set day=%date:~3,2%
set year=%date:~6,4%
echo The current month is %month%
echo The current day is %day%
echo The current year is %year%
pause >nul

Another option is the for command, again here is my example:

@echo off
for /f "delims=/ tokens=1-3" %%a in ("%date%") do (
set month=%%a
set day=%%b
set year=%%c
)
echo The current month is %month%
echo The current day is %day%
echo The current year is %year%
pause >nul

Order a MySQL table by two columns

ORDER BY article_rating ASC , article_time DESC

DESC at the end will sort by both columns descending. You have to specify ASC if you want it otherwise

Convert String to Float in Swift

Works on Swift 5+

import Foundation

let myString:String = "50"
let temp = myString as NSString
let myFloat = temp.floatValue
print(myFloat)  //50.0
print(type(of: myFloat)) // Float

// Also you can guard your value in order to check what is happening whenever your app crashes.

guard let myFloat = temp.floatValue else {
fatalError(" fail to change string to float value.")
}

Start ssh-agent on login

Tried couple solutions from many sources but all seemed like too much trouble. Finally I found the easiest one :)

If you're not yet familiar with zsh and oh-my-zsh then install it. You will love it :)

Then edit .zshrc

vim ~/.zshrc

find plugins section and update it to use ssh-agent like so:

plugins=(ssh-agent git)

And that's all! You'll have ssh-agent up and running every time you start your shell

What exactly is an instance in Java?

I think that Object = Instance. Reference is a "link" to an Object.

Car c = new Car();

variable c stores a reference to an object of type Car.

Understanding Matlab FFT example

There are some misconceptions here.

Frequencies above 500 can be represented in an FFT result of length 1000. Unfortunately these frequencies are all folded together and mixed into the first 500 FFT result bins. So normally you don't want to feed an FFT a signal containing any frequencies at or above half the sampling rate, as the FFT won't care and will just mix the high frequencies together with the low ones (aliasing) making the result pretty much useless. That's why data should be low-pass filtered before being sampled and fed to an FFT.

The FFT returns amplitudes without frequencies because the frequencies depend, not just on the length of the FFT, but also on the sample rate of the data, which isn't part of the FFT itself or it's input. You can feed the same length FFT data at any sample rate, as thus get any range of frequencies out of it.

The reason the result plots ends at 500 is that, for any real data input, the frequencies above half the length of the FFT are just mirrored repeats (complex conjugated) of the data in the first half. Since they are duplicates, most people just ignore them. Why plot duplicates? The FFT calculates the other half of the result for people who feed the FFT complex data (with both real and imaginary components), which does create two different halves.

How do I make a composite key with SQL Server Management Studio?

enter image description here

  1. Open the design table tab
  2. Highlight your two INT fields (Ctrl/Shift+click on the grey blocks in the very first column)
  3. Right click -> Set primary key

How to execute a .sql script from bash

You simply need to start mysql and feed it with the content of db.sql:

mysql -u user -p < db.sql

Find if value in column A contains value from column B?

You could try this

=IF(ISNA(VLOOKUP(<single column I value>,<entire column E range>,1,FALSE)),FALSE, TRUE)

-or-

=IF(ISNA(VLOOKUP(<single column I value>,<entire column E range>,1,FALSE)),"FALSE", "File found in row "   & MATCH(<single column I value>,<entire column E range>,0))

you could replace <single column I value> and <entire column E range> with named ranged. That'd probably be the easiest.

Just drag that formula all the way down the length of your I column in whatever column you want.

Query to display all tablespaces in a database and datafiles

In oracle, generally speaking, there are number of facts that I will mention in following section:

  • Each database can have many Schema/User (Logical division).
  • Each database can have many tablespaces (Logical division).
  • A schema is the set of objects (tables, indexes, views, etc) that belong to a user.
  • In Oracle, a user can be considered the same as a schema.
  • A database is divided into logical storage units called tablespaces, which group related logical structures together. For example, tablespaces commonly group all of an application’s objects to simplify some administrative operations. You may have a tablespace for application data and an additional one for application indexes.

Therefore, your question, "to see all tablespaces and datafiles belong to SCOTT" is s bit wrong.

However, there are some DBA views encompass information about all database objects, regardless of the owner. Only users with DBA privileges can access these views: DBA_DATA_FILES, DBA_TABLESPACES, DBA_FREE_SPACE, DBA_SEGMENTS.

So, connect to your DB as sysdba and run query through these helpful views. For example this query can help you to find all tablespaces and their data files that objects of your user are located:

SELECT DISTINCT sgm.TABLESPACE_NAME , dtf.FILE_NAME
FROM DBA_SEGMENTS sgm
JOIN DBA_DATA_FILES dtf ON (sgm.TABLESPACE_NAME = dtf.TABLESPACE_NAME)
WHERE sgm.OWNER = 'SCOTT'

How to clear memory to prevent "out of memory error" in excel vba?

Found this thread looking for a solution to my problem. Mine required a different solution that I figured out that might be of use to others. My macro was deleting rows, shifting up, and copying rows to another worksheet. Memory usage was exploding to several gigs and causing "out of memory" after processing around only 4000 records. What solved it for me?

application.screenupdating = false

Added that at the beginning of my code (be sure to make it true again, at the end) I knew that would make it run faster, which it did.. but had no idea about the memory thing.

After making this small change the memory usage didn't exceed 135 mb. Why did that work? No idea really. But it's worth a shot and might apply to you.

Error:Execution failed for task ':app:compileDebugKotlin'. > Compilation error. See log for more details

I changed my project to use Androidx, so I used the migration tool but some files(many files), didn't change automatically. I opened each file (activities, enums, fragments) and I found so many errors. I corrected them but the compile still show me incomprehensible errors. After looking for a solution I found this answer that someone said:

go to Analyze >> Inspect code

enter image description here

Whole Project:

enter image description here

It took some time and then showed me the result below:

enter image description here

As I corrected the errors I thought were important, I was running the build until the remaining errors were no longer affecting the build.

My Android Studio details

enter image description here

How many bits or bytes are there in a character?

It depends what is the character and what encoding it is in:

  • An ASCII character in 8-bit ASCII encoding is 8 bits (1 byte), though it can fit in 7 bits.

  • An ISO-8895-1 character in ISO-8859-1 encoding is 8 bits (1 byte).

  • A Unicode character in UTF-8 encoding is between 8 bits (1 byte) and 32 bits (4 bytes).

  • A Unicode character in UTF-16 encoding is between 16 (2 bytes) and 32 bits (4 bytes), though most of the common characters take 16 bits. This is the encoding used by Windows internally.

  • A Unicode character in UTF-32 encoding is always 32 bits (4 bytes).

  • An ASCII character in UTF-8 is 8 bits (1 byte), and in UTF-16 - 16 bits.

  • The additional (non-ASCII) characters in ISO-8895-1 (0xA0-0xFF) would take 16 bits in UTF-8 and UTF-16.

That would mean that there are between 0.03125 and 0.125 characters in a bit.

Convert a float64 to an int in Go

Simply casting to an int truncates the float, which if your system internally represent 2.0 as 1.9999999999, you will not get what you expect. The various printf conversions deal with this and properly round the number when converting. So to get a more accurate value, the conversion is even more complicated than you might first expect:

package main

import (
    "fmt"
    "strconv"
)

func main() {
    floats := []float64{1.9999, 2.0001, 2.0}
    for _, f := range floats {
        t := int(f)
        s := fmt.Sprintf("%.0f", f)
        if i, err := strconv.Atoi(s); err == nil {
            fmt.Println(f, t, i)
        } else {
            fmt.Println(f, t, err)
        }
    }
}

Code on Go Playground

What is the best way to do a substring in a batch file?

As an additional info to Joey's answer, which isn't described in the help of set /? nor for /?.

%~0 expands to the name of the own batch, exactly as it was typed.
So if you start your batch it will be expanded as

%~0   - mYbAtCh
%~n0  - mybatch
%~nx0 - mybatch.bat

But there is one exception, expanding in a subroutine could fail

echo main- %~0
call :myFunction
exit /b

:myFunction
echo func - %~0
echo func - %~n0
exit /b

This results to

main - myBatch
Func - :myFunction
func - mybatch

In a function %~0 expands always to the name of the function, not of the batch file.
But if you use at least one modifier it will show the filename again!

Check orientation on Android phone

i think using getRotationv() doesn't help because http://developer.android.com/reference/android/view/Display.html#getRotation%28%29 getRotation() Returns the rotation of the screen from its "natural" orientation.

so unless you know the "natural" orientation, rotation is meaningless.

i found an easier way,

  Display display = ((WindowManager) context.getSystemService(Context.WINDOW_SERVICE)).getDefaultDisplay();
  Point size = new Point();
  display.getSize(size);
  int width = size.x;
  int height = size.y;
  if(width>height)
    // its landscape

please tell me if there is a problem with this someone?

How to make multiple divs display in one line but still retain width?

I used the property

display: table;

and

display: table-cell;

to achieve the same.Link to fiddle below shows 3 tables wrapped in divs and these divs are further wrapped in a parent div

<div id='content'>
   <div id='div-1'><!-- COntains table --></div>
   <div id='div-2'><!-- contains two more divs that require to be arranged one below other --></div>
</div>

Here is the jsfiddle: http://jsfiddle.net/vikikamath/QU6WP/1/ I thought this might be helpful to someone looking to set divs in same line without using display-inline

Fast and simple String encrypt/decrypt in JAVA

Simplest way is to add this JAVA library using Gradle:

compile 'se.simbio.encryption:library:2.0.0'

You can use it as simple as this:

Encryption encryption = Encryption.getDefault("Key", "Salt", new byte[16]);
String encrypted = encryption.encryptOrNull("top secret string");
String decrypted = encryption.decryptOrNull(encrypted);

How to set shape's opacity?

In general you just have to define a slightly transparent color when creating the shape.

You can achieve that by setting the colors alpha channel.

#FF000000 will get you a solid black whereas #00000000 will get you a 100% transparent black (well it isn't black anymore obviously).

The color scheme is like this #AARRGGBB there A stands for alpha channel, R stands for red, G for green and B for blue.

The same thing applies if you set the color in Java. There it will only look like 0xFF000000.

UPDATE

In your case you'd have to add a solid node. Like below.

<?xml version="1.0" encoding="UTF-8"?>
<shape xmlns:android="http://schemas.android.com/apk/res/android"
    android:id="@+id/shape_my">
    <stroke android:width="4dp" android:color="#636161" />
    <padding android:left="20dp"
        android:top="20dp"
        android:right="20dp"
        android:bottom="20dp" />
    <corners android:radius="24dp" />
    <solid android:color="#88000000" />
</shape>

The color here is a half transparent black.

Where is svcutil.exe in Windows 7?

I don't think it is very important to find the location of Svcutil.exe. You can use Visual Studio Command prompt to execute directly without its absolute path,

Syntax:
svcutil.exe /language:[vb|cs] /out:[YourClassName].[cs|vb] /config:[YourAppConfigFile.config] [YourServiceAddress]

example:
svcutil.exe /language:cs /out:MyClientClass.cs /config:app.config http://localhost:8370/MyService/

Java escape JSON String?

The best method would be using some JSON library, e.g. Jackson ( http://jackson.codehaus.org ).

But if this is not an option simply escape msget before adding it to your string:

The wrong way to do this is

String msgetEscaped = msget.replaceAll("\"", "\\\"");

Either use (as recommended in the comments)

String msgetEscaped = msget.replace("\"", "\\\"");

or

String msgetEscaped = msget.replaceAll("\"", "\\\\\"");

A sample with all three variants can be found here: http://ideone.com/Nt1XzO

Remove Sub String by using Python

>>> import re
>>> st = " i think mabe 124 + <font color=\"black\"><font face=\"Times New Roman\">but I don't have a big experience it just how I see it in my eyes <font color=\"green\"><font face=\"Arial\">fun stuff"
>>> re.sub("<.*?>","",st)
" i think mabe 124 + but I don't have a big experience it just how I see it in my eyes fun stuff"
>>> 

How to Select Top 100 rows in Oracle?

you should use rownum in oracle to do what you seek

where rownum <= 100

see also those answers to help you

limit in oracle

select top in oracle

select top in oracle 2

dyld: Library not loaded: /usr/local/lib/libpng16.16.dylib with anything php related

I solved this by copying it over to the missing directory:

cp /opt/X11/lib/libpng15.15.dylib /usr/local/lib/libpng15.15.dylib

brew reinstall libpng kept installing libpng16, not libpng15 so I was forced to do the above.

What are the obj and bin folders (created by Visual Studio) used for?

I would encourage you to see this youtube video which demonstrates the difference between C# bin and obj folders and also explains how we get the benefit of incremental/conditional compilation.

C# compilation is a two-step process, see the below diagram for more details:

  1. Compiling: In compiling phase individual C# code files are compiled into individual compiled units. These individual compiled code files go in the OBJ directory.
  2. Linking: In the linking phase these individual compiled code files are linked to create single unit DLL and EXE. This goes in the BIN directory.

C# bin vs obj folders

If you compare both bin and obj directory you will find greater number of files in the "obj" directory as it has individual compiled code files while "bin" has a single unit.

bin vs obj

Html.HiddenFor value property not getting set

Simple way

@{
   Model.CRN = ViewBag.CRN;
}

@Html.HiddenFor(x => x.CRN)

Rounding a double to turn it into an int (java)

If you don't like Math.round() you can use this simple approach as well:

int a = (int) (doubleVar + 0.5);

How to revert initial git commit?

You can't. So:

rm -rf .git/
git init
git add -A
git commit -m 'Your new commit message'

Set focus on textbox in WPF

Another possible solution is to use FocusBehavior provided by free DevExpress MVVM Framework:

<TextBox Text="This control is focused on startup">
    <dxmvvm:Interaction.Behaviors>
        <dxmvvm:FocusBehavior/>
    </dxmvvm:Interaction.Behaviors>
</TextBox>

It allows you to focus a control when it's loaded, when a certain event is raised or a property is changed.

How to edit Docker container files from the host?

There are two ways to mount files into your container. It looks like you want a bind mount.

Bind Mounts

This mounts local files directly into the container's filesystem. The containerside path and the hostside path both point to the same file. Edits made from either side will show up on both sides.

  • mount the file:
? echo foo > ./foo
? docker run --mount type=bind,source=$(pwd)/foo,target=/foo -it debian:latest
# cat /foo
foo # local file shows up in container
  • in a separate shell, edit the file:
? echo 'bar' > ./foo # make a hostside change
  • back in the container:
# cat /foo
bar # the hostside change shows up
# echo baz > /foo # make a containerside change
# exit
 
? cat foo
baz # the containerside change shows up

Volume Mounts

  • mount the volume
? docker run --mount type=volume,source=foovolume,target=/foo  -it debian:latest
root@containerB# echo 'this is in a volume' > /foo/data
  • the local filesystem is unchanged
  • docker sees a new volume:
? docker volume ls
DRIVER    VOLUME NAME
local     foovolume
  • create a new container with the same volume
? docker run --mount type=volume,source=foovolume,target=/foo  -it debian:latest
root@containerC:/# cat /foo/data
this is in a volume # data is still available

syntax: -v vs --mount

These do the same thing. -v is more concise, --mount is more explicit.

bind mounts

-v /hostside/path:/containerside/path
--mount type=bind,source=/hostside/path,target=/containerside/path

volume mounts

-v /containerside/path
-v volumename:/containerside/path
--mount type=volume,source=volumename,target=/containerside/path

(If a volume name is not specified, a random one is chosen.)

The documentaion tries to convince you to use one thing in favor of another instead of just telling you how it works, which is confusing.

Replace Both Double and Single Quotes in Javascript String

mystring = mystring.replace(/["']/g, "");

Passing multiple parameters with $.ajax url

why not just pass an data an object with your key/value pairs then you don't have to worry about encoding

$.ajax({
    type: "Post",
    url: "getdata.php",
    data:{
       timestamp: timestamp,
       uid: id,
       uname: name
    },
    async: true,
    cache: false,
    success: function(data) {


    };
}?);?

Return value from a VBScript function

To return a value from a VBScript function, assign the value to the name of the function, like this:

Function getNumber
    getNumber = "423"
End Function

jQuery: how to get which button was clicked upon form submission?

I asked this same question: How can I get the button that caused the submit from the form submit event?

I ended up coming up with this solution and it worked pretty well:

$(document).ready(function() {
    $("form").submit(function() { 
        var val = $("input[type=submit][clicked=true]").val();
        // DO WORK
    });
    $("form input[type=submit]").click(function() {
        $("input[type=submit]", $(this).parents("form")).removeAttr("clicked");
        $(this).attr("clicked", "true");
    });
});

In your case with multiple forms you may need to tweak this a bit but it should still apply

Appending HTML string to the DOM

Quick Hack:


<script>
document.children[0].innerHTML="<h1>QUICK_HACK</h1>";
</script>

Use Cases:

1: Save as .html file and run in chrome or firefox or edge. (IE wont work)

2: Use in http://js.do

In Action: http://js.do/HeavyMetalCookies/quick_hack

Broken down with comments:

<script>

//: The message "QUICK_HACK" 
//: wrapped in a header #1 tag.
var text = "<h1>QUICK_HACK</h1>";

//: It's a quick hack, so I don't
//: care where it goes on the document,
//: just as long as I can see it.
//: Since I am doing this quick hack in
//: an empty file or scratchpad, 
//: it should be visible.
var child = document.children[0];

//: Set the html content of your child
//: to the message you want to see on screen.
child.innerHTML = text;

</script>

Reason Why I posted:

JS.do has two must haves:

  1. No autocomplete
  2. Vertical monitor friendly

But doesn't show console.log messages. Came here looking for a quick solution. I just want to see the results of a few lines of scratchpad code, the other solutions are too much work.

VB.Net: Dynamically Select Image from My.Resources

This works for me at runtime too:

UltraPictureBox1.Image = My.Resources.MyPicture

No strings involved and if I change the name it is automatically updated by refactoring.

Button button = findViewById(R.id.button) always resolves to null in Android Studio

The button code should be moved to the PlaceholderFragment() class. There you will call the layout fragment_main.xml in the onCreateView method. Like so

@Override
public View onCreateView(LayoutInflater inflater, ViewGroup container,
        Bundle savedInstanceState) {
    View view = inflater.inflate(R.layout.fragment_main, container, false);
    Button buttonClick = (Button) view.findViewById(R.id.button);
    buttonClick.setOnClickListener(new View.OnClickListener() {
        @Override
        public void onClick(View view) {
            onButtonClick((Button) view);
        }

    });

    return view;
}

Centering a background image, using CSS

There's an error in your code. You're using a mix of full syntax and shorthand notation on the background-image property.This is causing the no-repeat to be ignored, since it's not a valid value for the background-image property.

body{   
    background-position:center;
    background-image:url(../images/images2.jpg) no-repeat;
}

Should become one of the following:

body{
    background:url(../images/images2.jpg) center center no-repeat;
}

or

body
{
    background-image: url(path-to-file/img.jpg);
    background-repeat:no-repeat;
    background-position: center center;
}

EDIT: For your image 'scaling' issue, you might want to look at this question's answer.

Get values from other sheet using VBA

Usually I use this code (into a VBA macro) for getting a cell's value from another cell's value from another sheet:

Range("Y3") = ActiveWorkbook.Worksheets("Reference").Range("X4")

The cell Y3 is into a sheet that I called it "Calculate" The cell X4 is into a sheet that I called it "Reference" The VBA macro has been run when the "Calculate" in active sheet.

Vertical align text in block element

You can also try

a {
  height:100px;
  line-height:100px;
}

Throw HttpResponseException or return Request.CreateErrorResponse?

The approach I have taken is to just throw exceptions from the api controller actions and have an exception filter registered that processes the exception and sets an appropriate response on the action execution context.

The filter exposes a fluent interface that provides a means of registering handlers for specific types of exceptions prior to registering the filter with global configuration.

The use of this filter enables centralized exception handling instead of spreading it across the controller actions. There are however cases where I will catch exceptions within the controller action and return a specific response if it does not make sense to centralize the handling of that particular exception.

Example registration of filter:

GlobalConfiguration.Configuration.Filters.Add(
    new UnhandledExceptionFilterAttribute()
    .Register<KeyNotFoundException>(HttpStatusCode.NotFound)

    .Register<SecurityException>(HttpStatusCode.Forbidden)

    .Register<SqlException>(
        (exception, request) =>
        {
            var sqlException = exception as SqlException;

            if (sqlException.Number > 50000)
            {
                var response            = request.CreateResponse(HttpStatusCode.BadRequest);
                response.ReasonPhrase   = sqlException.Message.Replace(Environment.NewLine, String.Empty);

                return response;
            }
            else
            {
                return request.CreateResponse(HttpStatusCode.InternalServerError);
            }
        }
    )
);

UnhandledExceptionFilterAttribute class:

using System;
using System.Collections.Concurrent;
using System.Net;
using System.Net.Http;
using System.Text;
using System.Web.Http.Filters;

namespace Sample
{
    /// <summary>
    /// Represents the an attribute that provides a filter for unhandled exceptions.
    /// </summary>
    public class UnhandledExceptionFilterAttribute : ExceptionFilterAttribute
    {
        #region UnhandledExceptionFilterAttribute()
        /// <summary>
        /// Initializes a new instance of the <see cref="UnhandledExceptionFilterAttribute"/> class.
        /// </summary>
        public UnhandledExceptionFilterAttribute() : base()
        {

        }
        #endregion

        #region DefaultHandler
        /// <summary>
        /// Gets a delegate method that returns an <see cref="HttpResponseMessage"/> 
        /// that describes the supplied exception.
        /// </summary>
        /// <value>
        /// A <see cref="Func{Exception, HttpRequestMessage, HttpResponseMessage}"/> delegate method that returns 
        /// an <see cref="HttpResponseMessage"/> that describes the supplied exception.
        /// </value>
        private static Func<Exception, HttpRequestMessage, HttpResponseMessage> DefaultHandler = (exception, request) =>
        {
            if(exception == null)
            {
                return null;
            }

            var response            = request.CreateResponse<string>(
                HttpStatusCode.InternalServerError, GetContentOf(exception)
            );
            response.ReasonPhrase   = exception.Message.Replace(Environment.NewLine, String.Empty);

            return response;
        };
        #endregion

        #region GetContentOf
        /// <summary>
        /// Gets a delegate method that extracts information from the specified exception.
        /// </summary>
        /// <value>
        /// A <see cref="Func{Exception, String}"/> delegate method that extracts information 
        /// from the specified exception.
        /// </value>
        private static Func<Exception, string> GetContentOf = (exception) =>
        {
            if (exception == null)
            {
                return String.Empty;
            }

            var result  = new StringBuilder();

            result.AppendLine(exception.Message);
            result.AppendLine();

            Exception innerException = exception.InnerException;
            while (innerException != null)
            {
                result.AppendLine(innerException.Message);
                result.AppendLine();
                innerException = innerException.InnerException;
            }

            #if DEBUG
            result.AppendLine(exception.StackTrace);
            #endif

            return result.ToString();
        };
        #endregion

        #region Handlers
        /// <summary>
        /// Gets the exception handlers registered with this filter.
        /// </summary>
        /// <value>
        /// A <see cref="ConcurrentDictionary{Type, Tuple}"/> collection that contains 
        /// the exception handlers registered with this filter.
        /// </value>
        protected ConcurrentDictionary<Type, Tuple<HttpStatusCode?, Func<Exception, HttpRequestMessage, HttpResponseMessage>>> Handlers
        {
            get
            {
                return _filterHandlers;
            }
        }
        private readonly ConcurrentDictionary<Type, Tuple<HttpStatusCode?, Func<Exception, HttpRequestMessage, HttpResponseMessage>>> _filterHandlers = new ConcurrentDictionary<Type, Tuple<HttpStatusCode?, Func<Exception, HttpRequestMessage, HttpResponseMessage>>>();
        #endregion

        #region OnException(HttpActionExecutedContext actionExecutedContext)
        /// <summary>
        /// Raises the exception event.
        /// </summary>
        /// <param name="actionExecutedContext">The context for the action.</param>
        public override void OnException(HttpActionExecutedContext actionExecutedContext)
        {
            if(actionExecutedContext == null || actionExecutedContext.Exception == null)
            {
                return;
            }

            var type    = actionExecutedContext.Exception.GetType();

            Tuple<HttpStatusCode?, Func<Exception, HttpRequestMessage, HttpResponseMessage>> registration = null;

            if (this.Handlers.TryGetValue(type, out registration))
            {
                var statusCode  = registration.Item1;
                var handler     = registration.Item2;

                var response    = handler(
                    actionExecutedContext.Exception.GetBaseException(), 
                    actionExecutedContext.Request
                );

                // Use registered status code if available
                if (statusCode.HasValue)
                {
                    response.StatusCode = statusCode.Value;
                }

                actionExecutedContext.Response  = response;
            }
            else
            {
                // If no exception handler registered for the exception type, fallback to default handler
                actionExecutedContext.Response  = DefaultHandler(
                    actionExecutedContext.Exception.GetBaseException(), actionExecutedContext.Request
                );
            }
        }
        #endregion

        #region Register<TException>(HttpStatusCode statusCode)
        /// <summary>
        /// Registers an exception handler that returns the specified status code for exceptions of type <typeparamref name="TException"/>.
        /// </summary>
        /// <typeparam name="TException">The type of exception to register a handler for.</typeparam>
        /// <param name="statusCode">The HTTP status code to return for exceptions of type <typeparamref name="TException"/>.</param>
        /// <returns>
        /// This <see cref="UnhandledExceptionFilterAttribute"/> after the exception handler has been added.
        /// </returns>
        public UnhandledExceptionFilterAttribute Register<TException>(HttpStatusCode statusCode) 
            where TException : Exception
        {

            var type    = typeof(TException);
            var item    = new Tuple<HttpStatusCode?, Func<Exception, HttpRequestMessage, HttpResponseMessage>>(
                statusCode, DefaultHandler
            );

            if (!this.Handlers.TryAdd(type, item))
            {
                Tuple<HttpStatusCode?, Func<Exception, HttpRequestMessage, HttpResponseMessage>> oldItem = null;

                if (this.Handlers.TryRemove(type, out oldItem))
                {
                    this.Handlers.TryAdd(type, item);
                }
            }

            return this;
        }
        #endregion

        #region Register<TException>(Func<Exception, HttpRequestMessage, HttpResponseMessage> handler)
        /// <summary>
        /// Registers the specified exception <paramref name="handler"/> for exceptions of type <typeparamref name="TException"/>.
        /// </summary>
        /// <typeparam name="TException">The type of exception to register the <paramref name="handler"/> for.</typeparam>
        /// <param name="handler">The exception handler responsible for exceptions of type <typeparamref name="TException"/>.</param>
        /// <returns>
        /// This <see cref="UnhandledExceptionFilterAttribute"/> after the exception <paramref name="handler"/> 
        /// has been added.
        /// </returns>
        /// <exception cref="ArgumentNullException">The <paramref name="handler"/> is <see langword="null"/>.</exception>
        public UnhandledExceptionFilterAttribute Register<TException>(Func<Exception, HttpRequestMessage, HttpResponseMessage> handler) 
            where TException : Exception
        {
            if(handler == null)
            {
              throw new ArgumentNullException("handler");
            }

            var type    = typeof(TException);
            var item    = new Tuple<HttpStatusCode?, Func<Exception, HttpRequestMessage, HttpResponseMessage>>(
                null, handler
            );

            if (!this.Handlers.TryAdd(type, item))
            {
                Tuple<HttpStatusCode?, Func<Exception, HttpRequestMessage, HttpResponseMessage>> oldItem = null;

                if (this.Handlers.TryRemove(type, out oldItem))
                {
                    this.Handlers.TryAdd(type, item);
                }
            }

            return this;
        }
        #endregion

        #region Unregister<TException>()
        /// <summary>
        /// Unregisters the exception handler for exceptions of type <typeparamref name="TException"/>.
        /// </summary>
        /// <typeparam name="TException">The type of exception to unregister handlers for.</typeparam>
        /// <returns>
        /// This <see cref="UnhandledExceptionFilterAttribute"/> after the exception handler 
        /// for exceptions of type <typeparamref name="TException"/> has been removed.
        /// </returns>
        public UnhandledExceptionFilterAttribute Unregister<TException>()
            where TException : Exception
        {
            Tuple<HttpStatusCode?, Func<Exception, HttpRequestMessage, HttpResponseMessage>> item = null;

            this.Handlers.TryRemove(typeof(TException), out item);

            return this;
        }
        #endregion
    }
}

Source code can also be found here.

VSCode single to double quote automatic replace

quote_type = single

add this inside .editorconfig

# EditorConfig is awesome: https://EditorConfig.org

# top-most EditorConfig file
root = true

[*]
indent_style = space
indent_size = 2
end_of_line = lf
charset = utf-8
trim_trailing_whitespace = false
insert_final_newline = false
quote_type = single

Powershell v3 Invoke-WebRequest HTTPS error

Invoke-WebRequest "DomainName" -SkipCertificateCheck

You can use -SkipCertificateCheck Parameter to achieve this as a one-liner command ( THIS PARAMETER IS ONLY SUPPORTED ON CORE PSEDITION )

Changing SQL Server collation to case insensitive from case sensitive?

You can do that but the changes will affect for new data that is inserted on the database. On the long run follow as suggested above.

Also there are certain tricks you can override the collation, such as parameters for stored procedures or functions, alias data types, and variables are assigned the default collation of the database. To change the collation of an alias type, you must drop the alias and re-create it.

You can override the default collation of a literal string by using the COLLATE clause. If you do not specify a collation, the literal is assigned the database default collation. You can use DATABASEPROPERTYEX to find the current collation of the database.

You can override the server, database, or column collation by specifying a collation in the ORDER BY clause of a SELECT statement.

how to get data from selected row from datagridview

I was having the same issue and this works excellently.

Private Sub DataGridView17_CellFormatting(sender As Object, e As System.Windows.Forms.DataGridViewCellFormattingEventArgs) Handles DataGridView17.CellFormatting  
  'Display complete contents in tooltip even though column display cuts off part of it.   
  DataGridView17.Rows(e.RowIndex).Cells(e.ColumnIndex).ToolTipText = DataGridView17.Rows(e.RowIndex).Cells(e.ColumnIndex).Value 
End Sub

Text editor to open big (giant, huge, large) text files

Tips and tricks

less

Why are you using editors to just look at a (large) file?

Under *nix or Cygwin, just use less. (There is a famous saying – "less is more, more or less" – because "less" replaced the earlier Unix command "more", with the addition that you could scroll back up.) Searching and navigating under less is very similar to Vim, but there is no swap file and little RAM used.

There is a Win32 port of GNU less. See the "less" section of the answer above.

Perl

Perl is good for quick scripts, and its .. (range flip-flop) operator makes for a nice selection mechanism to limit the crud you have to wade through.

For example:

$ perl -n -e 'print if ( 1000000 .. 2000000)' humongo.txt | less

This will extract everything from line 1 million to line 2 million, and allow you to sift the output manually in less.

Another example:

$ perl -n -e 'print if ( /regex one/ .. /regex two/)' humongo.txt | less

This starts printing when the "regular expression one" finds something, and stops when the "regular expression two" find the end of an interesting block. It may find multiple blocks. Sift the output...

logparser

This is another useful tool you can use. To quote the Wikipedia article:

logparser is a flexible command line utility that was initially written by Gabriele Giuseppini, a Microsoft employee, to automate tests for IIS logging. It was intended for use with the Windows operating system, and was included with the IIS 6.0 Resource Kit Tools. The default behavior of logparser works like a "data processing pipeline", by taking an SQL expression on the command line, and outputting the lines containing matches for the SQL expression.

Microsoft describes Logparser as a powerful, versatile tool that provides universal query access to text-based data such as log files, XML files and CSV files, as well as key data sources on the Windows operating system such as the Event Log, the Registry, the file system, and Active Directory. The results of the input query can be custom-formatted in text based output, or they can be persisted to more specialty targets like SQL, SYSLOG, or a chart.

Example usage:

C:\>logparser.exe -i:textline -o:tsv "select Index, Text from 'c:\path\to\file.log' where line > 1000 and line < 2000"
C:\>logparser.exe -i:textline -o:tsv "select Index, Text from 'c:\path\to\file.log' where line like '%pattern%'"

The relativity of sizes

100 MB isn't too big. 3 GB is getting kind of big. I used to work at a print & mail facility that created about 2% of U.S. first class mail. One of the systems for which I was the tech lead accounted for about 15+% of the pieces of mail. We had some big files to debug here and there.

And more...

Feel free to add more tools and information here. This answer is community wiki for a reason! We all need more advice on dealing with large amounts of data...

How to convert int[] to Integer[] in Java?

Convert int[] to Integer[]:

    import java.util.Arrays;
    ...

    int[] aint = {1,2,3,4,5,6,7,8,9,10};
    Integer[] aInt = new Integer[aint.length];

    Arrays.setAll(aInt, i -> aint[i]);

convert nan value to zero

A code example for drake's answer to use nan_to_num:

>>> import numpy as np
>>> A = np.array([[1, 2, 3], [0, 3, np.NaN]])
>>> A = np.nan_to_num(A)
>>> A
array([[ 1.,  2.,  3.],
       [ 0.,  3.,  0.]])

Is there a concurrent List in Java's JDK?

There is a concurrent list implementation in java.util.concurrent. CopyOnWriteArrayList in particular.

"No rule to make target 'install'"... But Makefile exists

I also came across the same error. Here is the fix: If you are using Cmake-GUI:

  1. Clean the cache of the loaded libraries in Cmake-GUI File menu.
  2. Configure the libraries.
  3. Generate the Unix file.

If you missed the 3rd step:

*** No rule to make target `install'. Stop.

error will occur.

How do I remove the horizontal scrollbar in a div?

No scroll (without specifying x or y):

.your-class {
   overflow: hidden;
}

Remove horizontal scroll:

.your-class {
   overflow-x: hidden;
}

Remove vertical scroll:

.your-class {
   overflow-y: hidden;
}

Website screenshots

I used page2images. It is developed base on the cutycapt which is really fast and stable. If you do not want to spend too much time on the performance and configuration, you should use it. If you go to their website, you can find more details and sample PHP code.

Lumen: get URL parameter in a Blade view

You can publicly expose Input facade via an alias in config/app.php:

'aliases' => [
    ...

    'Input' => Illuminate\Support\Facades\Input::class,
]

And access url $_GET parameter values using the facade directly inside Blade view/template:

{{ Input::get('a') }}

Array or List in Java. Which is faster?

None of the answers had information that I was interested in - repetitive scan of the same array many many times. Had to create a JMH test for this.

Results (Java 1.8.0_66 x32, iterating plain array is at least 5 times quicker than ArrayList):

Benchmark                    Mode  Cnt   Score   Error  Units
MyBenchmark.testArrayForGet  avgt   10   8.121 ? 0.233  ms/op
MyBenchmark.testListForGet   avgt   10  37.416 ? 0.094  ms/op
MyBenchmark.testListForEach  avgt   10  75.674 ? 1.897  ms/op

Test

package my.jmh.test;

import java.util.ArrayList;
import java.util.List;
import java.util.concurrent.TimeUnit;
import org.openjdk.jmh.annotations.Benchmark;
import org.openjdk.jmh.annotations.BenchmarkMode;
import org.openjdk.jmh.annotations.Fork;
import org.openjdk.jmh.annotations.Measurement;
import org.openjdk.jmh.annotations.Mode;
import org.openjdk.jmh.annotations.OutputTimeUnit;
import org.openjdk.jmh.annotations.Scope;
import org.openjdk.jmh.annotations.State;
import org.openjdk.jmh.annotations.Warmup;

@State(Scope.Benchmark)
@Fork(1)
@Warmup(iterations = 5, timeUnit = TimeUnit.SECONDS)
@Measurement(iterations = 10)
@BenchmarkMode(Mode.AverageTime)
@OutputTimeUnit(TimeUnit.MILLISECONDS)
public class MyBenchmark {

    public final static int ARR_SIZE = 100;
    public final static int ITER_COUNT = 100000;

    String arr[] = new String[ARR_SIZE];
    List<String> list = new ArrayList<>(ARR_SIZE);

    public MyBenchmark() {
        for( int i = 0; i < ARR_SIZE; i++ ) {
            list.add(null);
        }
    }

    @Benchmark
    public void testListForEach() {
        int count = 0;
        for( int i = 0; i < ITER_COUNT; i++ ) {
            for( String str : list ) {
                if( str != null )
                    count++;
            }
        }
        if( count > 0 )
            System.out.print(count);
    }

    @Benchmark
    public void testListForGet() {
        int count = 0;
        for( int i = 0; i < ITER_COUNT; i++ ) {
            for( int j = 0; j < ARR_SIZE; j++ ) {
                if( list.get(j) != null )
                    count++;
            }
        }
        if( count > 0 )
            System.out.print(count);
    }

    @Benchmark
    public void testArrayForGet() {
        int count = 0;
        for( int i = 0; i < ITER_COUNT; i++ ) {
            for( int j = 0; j < ARR_SIZE; j++ ) {
                if( arr[j] != null )
                    count++;
            }
        }
        if( count > 0 )
            System.out.print(count);
    }

}

How do I create and access the global variables in Groovy?

Like all OO languages, Groovy has no concept of "global" by itself (unlike, say, BASIC, Python or Perl).

If you have several methods that need to share the same variable, use a field:

class Foo {
    def a;

    def foo() {
        a = 1;
    }
    def bar() {
        print a;
    }
}

Round up to Second Decimal Place in Python

Note that the ceil(num * 100) / 100 trick will crash on some degenerate inputs, like 1e308. This may not come up often but I can tell you it just cost me a couple of days. To avoid this, "it would be nice if" ceil() and floor() took a decimal places argument, like round() does... Meanwhile, anyone know a clean alternative that won't crash on inputs like this? I had some hopes for the decimal package but it seems to die too:

>>> from math import ceil
>>> from decimal import Decimal, ROUND_DOWN, ROUND_UP
>>> num = 0.1111111111000
>>> ceil(num * 100) / 100
0.12
>>> float(Decimal(num).quantize(Decimal('.01'), rounding=ROUND_UP))
0.12
>>> num = 1e308
>>> ceil(num * 100) / 100
Traceback (most recent call last):
  File "<string>", line 301, in runcode
  File "<interactive input>", line 1, in <module>
OverflowError: cannot convert float infinity to integer
>>> float(Decimal(num).quantize(Decimal('.01'), rounding=ROUND_UP))
Traceback (most recent call last):
  File "<string>", line 301, in runcode
  File "<interactive input>", line 1, in <module>
decimal.InvalidOperation: [<class 'decimal.InvalidOperation'>]

Of course one might say that crashing is the only sane behavior on such inputs, but I would argue that it's not the rounding but the multiplication that's causing the problem (that's why, eg, 1e306 doesn't crash), and a cleaner implementation of the round-up-nth-place fn would avoid the multiplication hack.

Moment.js with ReactJS (ES6)

If the other answers fail, importing it as

import moment from 'moment/moment.js'

may work.

Oracle: not a valid month

You can also change the value of this database parameter for your session by using the ALTER SESSION command and use it as you wanted

ALTER SESSION SET NLS_DATE_FORMAT = 'DD-MM-YYYY';
SELECT TO_DATE('05-12-2015') FROM dual;

05/12/2015

If using maven, usually you put log4j.properties under java or resources?

src/main/resources is the "standard placement" for this.

Update: The above answers the question, but its not the best solution. Check out the other answers and the comments on this ... you would probably not shipping your own logging properties with the jar but instead leave it to the client (for example app-server, stage environment, etc) to configure the desired logging. Thus, putting it in src/test/resources is my preferred solution.

Note: Speaking of leaving the concrete log config to the client/user, you should consider replacing log4j with slf4j in your app.

How to stop PHP code execution?

You can use __halt_compiler function which will Halt the compiler execution

http://www.php.net/manual/en/function.halt-compiler.php

Angular get object from array by Id

CASE - 1

Using array.filter() We can get an array of objects which will match with our condition.
see the working example.

_x000D_
_x000D_
var questions = [
      {id: 1, question: "Do you feel a connection to a higher source and have a sense of comfort knowing that you are part of something greater than yourself?", category: "Spiritual", subs: []},
      {id: 2, question: "Do you feel you are free of unhealthy behavior that impacts your overall well-being?", category: "Habits", subs: []},
      {id: 3, question: "1 Do you feel you have healthy and fulfilling relationships?", category: "Relationships", subs: []},
      {id: 3, question: "2 Do you feel you have healthy and fulfilling relationships?", category: "Relationships", subs: []},
      {id: 3, question: "3 Do you feel you have healthy and fulfilling relationships?", category: "Relationships", subs: []},
      {id: 4, question: "Do you feel you have a sense of purpose and that you have a positive outlook about yourself and life?", category: "Emotional Well-being", subs: []},
      {id: 5, question: "Do you feel you have a healthy diet and that you are fueling your body for optimal health? ", category: "Eating Habits ", subs: []},
      {id: 6, question: "Do you feel that you get enough rest and that your stress level is healthy?", category: "Relaxation ", subs: []},
      {id: 7, question: "Do you feel you get enough physical activity for optimal health?", category: "Exercise ", subs: []},
      {id: 8, question: "Do you feel you practice self-care and go to the doctor regularly?", category: "Medical Maintenance", subs: []},
      {id: 9, question: "Do you feel satisfied with your income and economic stability?", category: "Financial", subs: []},
      {id: 10, question: "1 Do you feel you do fun things and laugh enough in your life?", category: "Play", subs: []},
      {id: 10, question: "2 Do you feel you do fun things and laugh enough in your life?", category: "Play", subs: []},
      {id: 11, question: "Do you feel you have a healthy sense of balance in this area of your life?", category: "Work-life Balance", subs: []},
      {id: 12, question: "Do you feel a sense of peace and contentment  in your home? ", category: "Home Environment", subs: []},
      {id: 13, question: "Do you feel that you are challenged and growing as a person?", category: "Intellectual Wellbeing", subs: []},
      {id: 14, question: "Do you feel content with what you see when you look in the mirror?", category: "Self-image", subs: []},
      {id: 15, question: "Do you feel engaged at work and a sense of fulfillment with your job?", category: "Work Satisfaction", subs: []}
];

function filter(){
  console.clear();
  var filter_id = document.getElementById("filter").value;
  var filter_array = questions.filter(x => x.id == filter_id);
  console.log(filter_array);
}
_x000D_
button {
  background: #0095ff;
  color: white;
  border: none;
  border-radius: 3px;
  padding: 8px;
  cursor: pointer;
}

input {
  padding: 8px;
}
_x000D_
<div>
  <label for="filter"></label>
  <input id="filter" type="number" name="filter" placeholder="Enter id which you want to filter">
  <button onclick="filter()">Filter</button>
</div>
_x000D_
_x000D_
_x000D_

CASE - 2

Using array.find() we can get first matched item and break the iteration.

_x000D_
_x000D_
var questions = [
      {id: 1, question: "Do you feel a connection to a higher source and have a sense of comfort knowing that you are part of something greater than yourself?", category: "Spiritual", subs: []},
      {id: 2, question: "Do you feel you are free of unhealthy behavior that impacts your overall well-being?", category: "Habits", subs: []},
      {id: 3, question: "1 Do you feel you have healthy and fulfilling relationships?", category: "Relationships", subs: []},
      {id: 3, question: "2 Do you feel you have healthy and fulfilling relationships?", category: "Relationships", subs: []},
      {id: 3, question: "3 Do you feel you have healthy and fulfilling relationships?", category: "Relationships", subs: []},
      {id: 4, question: "Do you feel you have a sense of purpose and that you have a positive outlook about yourself and life?", category: "Emotional Well-being", subs: []},
      {id: 5, question: "Do you feel you have a healthy diet and that you are fueling your body for optimal health? ", category: "Eating Habits ", subs: []},
      {id: 6, question: "Do you feel that you get enough rest and that your stress level is healthy?", category: "Relaxation ", subs: []},
      {id: 7, question: "Do you feel you get enough physical activity for optimal health?", category: "Exercise ", subs: []},
      {id: 8, question: "Do you feel you practice self-care and go to the doctor regularly?", category: "Medical Maintenance", subs: []},
      {id: 9, question: "Do you feel satisfied with your income and economic stability?", category: "Financial", subs: []},
      {id: 10, question: "1 Do you feel you do fun things and laugh enough in your life?", category: "Play", subs: []},
      {id: 10, question: "2 Do you feel you do fun things and laugh enough in your life?", category: "Play", subs: []},
      {id: 11, question: "Do you feel you have a healthy sense of balance in this area of your life?", category: "Work-life Balance", subs: []},
      {id: 12, question: "Do you feel a sense of peace and contentment  in your home? ", category: "Home Environment", subs: []},
      {id: 13, question: "Do you feel that you are challenged and growing as a person?", category: "Intellectual Wellbeing", subs: []},
      {id: 14, question: "Do you feel content with what you see when you look in the mirror?", category: "Self-image", subs: []},
      {id: 15, question: "Do you feel engaged at work and a sense of fulfillment with your job?", category: "Work Satisfaction", subs: []}
];

function find(){
  console.clear();
  var find_id = document.getElementById("find").value;
  var find_object = questions.find(x => x.id == find_id);
  console.log(find_object);
}
_x000D_
button {
  background: #0095ff;
  color: white;
  border: none;
  border-radius: 3px;
  padding: 8px;
  cursor: pointer;
}

input {
  padding: 8px;
  width: 200px;
}
_x000D_
<div>
  <label for="find"></label>
  <input id="find" type="number" name="find" placeholder="Enter id which you want to find">
  <button onclick="find()">Find</button>
</div>
_x000D_
_x000D_
_x000D_

"This operation requires IIS integrated pipeline mode."

For Visual Studio 2012 while debugging that error accrued

Website Menu -> Use IIS Express did it for me

Conversion failed when converting from a character string to uniqueidentifier - Two GUIDs

You have to check unique identifier column and you have to give a diff value to that particular field if you give the same value it will not work. It enforces uniqueness of the key.

Here is the code:

Insert into production.product 
(Name,ProductNumber,MakeFlag,FinishedGoodsFlag,Color,SafetyStockLevel,ReorderPoint,StandardCost,ListPrice,Size
,SizeUnitMeasureCode,WeightUnitMeasureCode,Weight,DaysToManufacture,
    ProductLine, 
    Class, 
    Style ,
    ProductSubcategoryID 
    ,ProductModelID 
    ,SellStartDate 
,SellEndDate 
    ,DiscontinuedDate 
    ,rowguid
    ,ModifiedDate 
  )
  values ('LL lemon' ,'BC-1234',0,0,'blue',400,960,0.00,100.00,Null,Null,Null,null,1,null,null,null,null,null,'1998-06-01 00:00:00.000',null,null,'C4244F0C-ABCE-451B-A895-83C0E6D1F468','2004-03-11 10:01:36.827')

Python 3 - Encode/Decode vs Bytes/Str

Neither is better than the other, they do exactly the same thing. However, using .encode() and .decode() is the more common way to do it. It is also compatible with Python 2.

Remove the title bar in Windows Forms

Set FormsBorderStyle of the Form to None.

If you do, it's up to you how to implement the dragging and closing functionality of the window.

How to encode the filename parameter of Content-Disposition header in HTTP?

RFC 6266 describes the “Use of the Content-Disposition Header Field in the Hypertext Transfer Protocol (HTTP)”. Quoting from that:

6. Internationalization Considerations

The “filename*” parameter (Section 4.3), using the encoding defined in [RFC5987], allows the server to transmit characters outside the ISO-8859-1 character set, and also to optionally specify the language in use.

And in their examples section:

This example is the same as the one above, but adding the "filename" parameter for compatibility with user agents not implementing RFC 5987:

Content-Disposition: attachment;
                     filename="EURO rates";
                     filename*=utf-8''%e2%82%ac%20rates

Note: Those user agents that do not support the RFC 5987 encoding ignore “filename*” when it occurs after “filename”.

In Appendix D there is also a long list of suggestions to increase interoperability. It also points at a site which compares implementations. Current all-pass tests suitable for common file names include:

  • attwithisofnplain: plain ISO-8859-1 file name with double quotes and without encoding. This requires a file name which is all ISO-8859-1 and does not contain percent signs, at least not in front of hex digits.
  • attfnboth: two parameters in the order described above. Should work for most file names on most browsers, although IE8 will use the “filename” parameter.

That RFC 5987 in turn references RFC 2231, which describes the actual format. 2231 is primarily for mail, and 5987 tells us what parts may be used for HTTP headers as well. Don't confuse this with MIME headers used inside a multipart/form-data HTTP body, which is governed by RFC 2388 (section 4.4 in particular) and the HTML 5 draft.

Jquery AJAX: No 'Access-Control-Allow-Origin' header is present on the requested resource

If the requested resource of the server is using Flask. Install Flask-CORS.

How to add a new column to a CSV file?

Yes Its a old question but it might help some

import csv
import uuid

# read and write csv files
with open('in_file','r') as r_csvfile:
    with open('out_file','w',newline='') as w_csvfile:

        dict_reader = csv.DictReader(r_csvfile,delimiter='|')
        #add new column with existing
        fieldnames = dict_reader.fieldnames + ['ADDITIONAL_COLUMN']
        writer_csv = csv.DictWriter(w_csvfile,fieldnames,delimiter='|')
        writer_csv.writeheader()


        for row in dict_reader:
            row['ADDITIONAL_COLUMN'] = str(uuid.uuid4().int >> 64) [0:6]
            writer_csv.writerow(row)

ASP.NET Display "Loading..." message while update panel is updating

You can use code as below when

using Image as Loading

<asp:UpdateProgress id="updateProgress" runat="server">
    <ProgressTemplate>
        <div style="position: fixed; text-align: center; height: 100%; width: 100%; top: 0; right: 0; left: 0; z-index: 9999999; background-color: #000000; opacity: 0.7;">
            <asp:Image ID="imgUpdateProgress" runat="server" ImageUrl="~/images/ajax-loader.gif" AlternateText="Loading ..." ToolTip="Loading ..." style="padding: 10px;position:fixed;top:45%;left:50%;" />
        </div>
    </ProgressTemplate>
</asp:UpdateProgress>

using Text as Loading

<asp:UpdateProgress id="updateProgress" runat="server">
    <ProgressTemplate>
        <div style="position: fixed; text-align: center; height: 100%; width: 100%; top: 0; right: 0; left: 0; z-index: 9999999; background-color: #000000; opacity: 0.7;">
            <span style="border-width: 0px; position: fixed; padding: 50px; background-color: #FFFFFF; font-size: 36px; left: 40%; top: 40%;">Loading ...</span>
        </div>
    </ProgressTemplate>
</asp:UpdateProgress>

What is the difference between background and background-color

Comparison of 18 color swatches rendered 100 times on a page as small rectangles, once with background and once with background-color.

I recreated the CSS performance experiment and the results are significantly different nowadays.

background

Chrome 54: 443 (µs/div)

Firefox 49: 162 (µs/div)

Edge 10: 56 (µs/div)

background-color

Chrome 54: 449 (µs/div)

Firefox 49: 171 (µs/div)

Edge 10: 58 (µs/div)

As you see - there's almost no difference.

Efficient way to rotate a list in python

I take this cost model as a reference:

http://scripts.mit.edu/~6.006/fall07/wiki/index.php?title=Python_Cost_Model

Your method of slicing the list and concatenating two sub-lists are linear-time operations. I would suggest using pop, which is a constant-time operation, e.g.:

def shift(list, n):
    for i in range(n)
        temp = list.pop()
        list.insert(0, temp)

How do I create a round cornered UILabel on the iPhone?

In Monotouch / Xamarin.iOS I solved the same problem like this:

UILabel exampleLabel = new UILabel(new CGRect(0, 0, 100, 50))
        {
            Text = "Hello Monotouch red label"
        };
        exampleLabel.Layer.MasksToBounds = true;
        exampleLabel.Layer.CornerRadius = 8;
        exampleLabel.Layer.BorderColor = UIColor.Red.CGColor;
        exampleLabel.Layer.BorderWidth = 2;

Using jquery to delete all elements with a given id

The cleanest way to do it is by using html5 selectors api, specifically querySelectorAll().

var contentToRemove = document.querySelectorAll("#myid");
$(contentToRemove).remove(); 

The querySelectorAll() function returns an array of dom elements matching a specific id. Once you have assigned the returned array to a var, then you can pass it as an argument to jquery remove().

Create Log File in Powershell

function WriteLog
{
    Param ([string]$LogString)
    $LogFile = "C:\$(gc env:computername).log"
    $DateTime = "[{0:MM/dd/yy} {0:HH:mm:ss}]" -f (Get-Date)
    $LogMessage = "$Datetime $LogString"
    Add-content $LogFile -value $LogMessage
}

WriteLog "This is my log message"

Android: How to get a custom View's height and width?

I was also lost around getMeasuredWidth() and getMeasuredHeight() getHeight() and getWidth() for a long time.......... later i found onSizeChanged() method to be REALLY helpful.

New Blog Post: how to get width and height dimensions of a customView (extends View) in Android http://syedrakibalhasan.blogspot.com/2011/02/how-to-get-width-and-height-dimensions.html

Insert new column into table in sqlite?

You can add new column with the query

ALTER TABLE TableName ADD COLUMN COLNew CHAR(25)

But it will be added at the end, not in between the existing columns.

ECONNREFUSED error when connecting to mongodb from node.js

I also got stucked with same problem so I fixed it like this :

If you are running mongo and nodejs in docker container or in docker compose

so replace localhost with mongo (which is container name in docker in my case) something like this below in your nodejs mongo connection file.

var mongoURI = "mongodb://mongo:27017/<nodejs_container_name>";

Change one value based on another value in pandas

One option is to use Python's slicing and indexing features to logically evaluate the places where your condition holds and overwrite the data there.

Assuming you can load your data directly into pandas with pandas.read_csv then the following code might be helpful for you.

import pandas
df = pandas.read_csv("test.csv")
df.loc[df.ID == 103, 'FirstName'] = "Matt"
df.loc[df.ID == 103, 'LastName'] = "Jones"

As mentioned in the comments, you can also do the assignment to both columns in one shot:

df.loc[df.ID == 103, ['FirstName', 'LastName']] = 'Matt', 'Jones'

Note that you'll need pandas version 0.11 or newer to make use of loc for overwrite assignment operations.


Another way to do it is to use what is called chained assignment. The behavior of this is less stable and so it is not considered the best solution (it is explicitly discouraged in the docs), but it is useful to know about:

import pandas
df = pandas.read_csv("test.csv")
df['FirstName'][df.ID == 103] = "Matt"
df['LastName'][df.ID == 103] = "Jones"

Change status bar text color to light in iOS 9 with Objective-C

Add the key View controller-based status bar appearance to Info.plist file and make it boolean type set to NO.

Insert one line code in viewDidLoad (this works on specific class where it is mentioned)

 [UIApplication sharedApplication].statusBarStyle = UIStatusBarStyleLightContent;

How to open, read, and write from serial port in C?

For demo code that conforms to POSIX standard as described in Setting Terminal Modes Properly and Serial Programming Guide for POSIX Operating Systems, the following is offered.
This code should execute correctly using Linux on x86 as well as ARM (or even CRIS) processors.
It's essentially derived from the other answer, but inaccurate and misleading comments have been corrected.

This demo program opens and initializes a serial terminal at 115200 baud for non-canonical mode that is as portable as possible.
The program transmits a hardcoded text string to the other terminal, and delays while the output is performed.
The program then enters an infinite loop to receive and display data from the serial terminal.
By default the received data is displayed as hexadecimal byte values.

To make the program treat the received data as ASCII codes, compile the program with the symbol DISPLAY_STRING, e.g.

 cc -DDISPLAY_STRING demo.c

If the received data is ASCII text (rather than binary data) and you want to read it as lines terminated by the newline character, then see this answer for a sample program.


#define TERMINAL    "/dev/ttyUSB0"

#include <errno.h>
#include <fcntl.h> 
#include <stdio.h>
#include <stdlib.h>
#include <string.h>
#include <termios.h>
#include <unistd.h>

int set_interface_attribs(int fd, int speed)
{
    struct termios tty;

    if (tcgetattr(fd, &tty) < 0) {
        printf("Error from tcgetattr: %s\n", strerror(errno));
        return -1;
    }

    cfsetospeed(&tty, (speed_t)speed);
    cfsetispeed(&tty, (speed_t)speed);

    tty.c_cflag |= (CLOCAL | CREAD);    /* ignore modem controls */
    tty.c_cflag &= ~CSIZE;
    tty.c_cflag |= CS8;         /* 8-bit characters */
    tty.c_cflag &= ~PARENB;     /* no parity bit */
    tty.c_cflag &= ~CSTOPB;     /* only need 1 stop bit */
    tty.c_cflag &= ~CRTSCTS;    /* no hardware flowcontrol */

    /* setup for non-canonical mode */
    tty.c_iflag &= ~(IGNBRK | BRKINT | PARMRK | ISTRIP | INLCR | IGNCR | ICRNL | IXON);
    tty.c_lflag &= ~(ECHO | ECHONL | ICANON | ISIG | IEXTEN);
    tty.c_oflag &= ~OPOST;

    /* fetch bytes as they become available */
    tty.c_cc[VMIN] = 1;
    tty.c_cc[VTIME] = 1;

    if (tcsetattr(fd, TCSANOW, &tty) != 0) {
        printf("Error from tcsetattr: %s\n", strerror(errno));
        return -1;
    }
    return 0;
}

void set_mincount(int fd, int mcount)
{
    struct termios tty;

    if (tcgetattr(fd, &tty) < 0) {
        printf("Error tcgetattr: %s\n", strerror(errno));
        return;
    }

    tty.c_cc[VMIN] = mcount ? 1 : 0;
    tty.c_cc[VTIME] = 5;        /* half second timer */

    if (tcsetattr(fd, TCSANOW, &tty) < 0)
        printf("Error tcsetattr: %s\n", strerror(errno));
}


int main()
{
    char *portname = TERMINAL;
    int fd;
    int wlen;
    char *xstr = "Hello!\n";
    int xlen = strlen(xstr);

    fd = open(portname, O_RDWR | O_NOCTTY | O_SYNC);
    if (fd < 0) {
        printf("Error opening %s: %s\n", portname, strerror(errno));
        return -1;
    }
    /*baudrate 115200, 8 bits, no parity, 1 stop bit */
    set_interface_attribs(fd, B115200);
    //set_mincount(fd, 0);                /* set to pure timed read */

    /* simple output */
    wlen = write(fd, xstr, xlen);
    if (wlen != xlen) {
        printf("Error from write: %d, %d\n", wlen, errno);
    }
    tcdrain(fd);    /* delay for output */


    /* simple noncanonical input */
    do {
        unsigned char buf[80];
        int rdlen;

        rdlen = read(fd, buf, sizeof(buf) - 1);
        if (rdlen > 0) {
#ifdef DISPLAY_STRING
            buf[rdlen] = 0;
            printf("Read %d: \"%s\"\n", rdlen, buf);
#else /* display hex */
            unsigned char   *p;
            printf("Read %d:", rdlen);
            for (p = buf; rdlen-- > 0; p++)
                printf(" 0x%x", *p);
            printf("\n");
#endif
        } else if (rdlen < 0) {
            printf("Error from read: %d: %s\n", rdlen, strerror(errno));
        } else {  /* rdlen == 0 */
            printf("Timeout from read\n");
        }               
        /* repeat read to get full message */
    } while (1);
}

For an example of an efficient program that provides buffering of received data yet allows byte-by-byte handing of the input, then see this answer.


java how to use classes in other package?

You have to provide the full path that you want to import.

import com.my.stuff.main.Main;
import com.my.stuff.second.*;

So, in your main class, you'd have:

package com.my.stuff.main

import com.my.stuff.second.Second;   // THIS IS THE IMPORTANT LINE FOR YOUR QUESTION

class Main {
   public static void main(String[] args) {
      Second second = new Second();
      second.x();  
   }
}

EDIT: adding example in response to Shawn D's comment

There is another alternative, as Shawn D points out, where you can specify the full package name of the object that you want to use. This is very useful in two locations. First, if you're using the class exactly once:

class Main {
    void function() {
        int x = my.package.heirarchy.Foo.aStaticMethod();

        another.package.heirarchy.Baz b = new another.package.heirarchy.Bax();
    }
}

Alternatively, this is useful when you want to differentiate between two classes with the same short name:

class Main {
    void function() {
        java.util.Date utilDate = ...;
        java.sql.Date sqlDate = ...;
    }
}

Reduce git repository size

Thanks for your replies. Here's what I did:

git gc
git gc --aggressive
git prune

That seemed to have done the trick. I started with around 10.5MB and now it's little more than 980KBs.

Reading a huge .csv file

I do a fair amount of vibration analysis and look at large data sets (tens and hundreds of millions of points). My testing showed the pandas.read_csv() function to be 20 times faster than numpy.genfromtxt(). And the genfromtxt() function is 3 times faster than the numpy.loadtxt(). It seems that you need pandas for large data sets.

I posted the code and data sets I used in this testing on a blog discussing MATLAB vs Python for vibration analysis.

Using python's mock patch.object to change the return value of a method called within another method

There are two ways you can do this; with patch and with patch.object

Patch assumes that you are not directly importing the object but that it is being used by the object you are testing as in the following

#foo.py
def some_fn():
    return 'some_fn'

class Foo(object):
    def method_1(self):
        return some_fn()
#bar.py
import foo
class Bar(object):
    def method_2(self):
        tmp = foo.Foo()
        return tmp.method_1()
#test_case_1.py
import bar
from mock import patch

@patch('foo.some_fn')
def test_bar(mock_some_fn):
    mock_some_fn.return_value = 'test-val-1'
    tmp = bar.Bar()
    assert tmp.method_2() == 'test-val-1'
    mock_some_fn.return_value = 'test-val-2'
    assert tmp.method_2() == 'test-val-2'

If you are directly importing the module to be tested, you can use patch.object as follows:

#test_case_2.py
import foo
from mock import patch

@patch.object(foo, 'some_fn')
def test_foo(test_some_fn):
    test_some_fn.return_value = 'test-val-1'
    tmp = foo.Foo()
    assert tmp.method_1() == 'test-val-1'
    test_some_fn.return_value = 'test-val-2'
    assert tmp.method_1() == 'test-val-2'

In both cases some_fn will be 'un-mocked' after the test function is complete.

Edit: In order to mock multiple functions, just add more decorators to the function and add arguments to take in the extra parameters

@patch.object(foo, 'some_fn')
@patch.object(foo, 'other_fn')
def test_foo(test_other_fn, test_some_fn):
    ...

Note that the closer the decorator is to the function definition, the earlier it is in the parameter list.

Check if a string contains another string

There is also the InStrRev function which does the same type of thing, but starts searching from the end of the text to the beginning.

Per @rene's answer...

Dim pos As Integer
pos = InStrRev("find the comma, in the string", ",")

...would still return 15 to pos, but if the string has more than one of the search string, like the word "the", then:

Dim pos As Integer
pos = InStrRev("find the comma, in the string", "the")

...would return 20 to pos, instead of 6.

How to create a template function within a class? (C++)

See here: Templates, template methods,Member Templates, Member Function Templates

class   Vector
{
  int     array[3];

  template <class TVECTOR2> 
  void  eqAdd(TVECTOR2 v2);
};

template <class TVECTOR2>
void    Vector::eqAdd(TVECTOR2 a2)
{
  for (int i(0); i < 3; ++i) array[i] += a2[i];
}

Mysql where id is in array

Change

$array=array_map('intval', explode(',', $string));

To:

$array= implode(',', array_map('intval', explode(',', $string)));

array_map returns an array, not a string. You need to convert the array to a comma separated string in order to use in the WHERE clause.

VBA changing active workbook

Use ThisWorkbook which will refer to the original workbook which holds the code.

Alternatively at code start

Dim Wb As Workbook
Set Wb = ActiveWorkbook

sample code that activates all open books before returning to ThisWorkbook

Sub Test()
Dim Wb As Workbook
Dim Wb2 As Workbook
Set Wb = ThisWorkbook
For Each Wb2 In Application.Workbooks
    Wb2.Activate
Next
Wb.Activate
End Sub

Easy way to use variables of enum types as string in C?

If the enum index is 0-based, you can put the names in an array of char*, and index them with the enum value.

Change visibility of ASP.NET label with JavaScript

Try this.

<asp:Button id="myButton" runat="server" style="display:none" Text="Click Me" />

<script type="text/javascript">
    function ShowButton() {
        var buttonID = '<%= myButton.ClientID %>';
        var button = document.getElementById(buttonID);
        if(button) { button.style.display = 'inherit'; }
    }
</script>

Don't use server-side code to do this because that would require a postback. Instead of using Visibility="false", you can just set a CSS property that hides the button. Then, in javascript, switch that property back whenever you want to show the button again.

The ClientID is used because it can be different from the server ID if the button is inside a Naming Container control. These include Panels of various sorts.

Effect of using sys.path.insert(0, path) and sys.path(append) when loading modules

I'm quite a beginner in Python and I found the answer of Anand was very good but quite complicated to me, so I try to reformulate :

1) insert and append methods are not specific to sys.path and as in other languages they add an item into a list or array and :
* append(item) add item to the end of the list,
* insert(n, item) inserts the item at the nth position in the list (0 at the beginning, 1 after the first element, etc ...).

2) As Anand said, python search the import files in each directory of the path in the order of the path, so :
* If you have no file name collisions, the order of the path has no impact,
* If you look after a function already defined in the path and you use append to add your path, you will not get your function but the predefined one.

But I think that it is better to use append and not insert to not overload the standard behaviour of Python, and use non-ambiguous names for your files and methods.

RuntimeWarning: DateTimeField received a naive datetime

One can both fix the warning and use the timezone specified in settings.py, which might be different from UTC.

For example in my settings.py I have:

USE_TZ = True
TIME_ZONE = 'Europe/Paris'

Here is a solution; the advantage is that str(mydate) gives the correct time:

>>> from datetime import datetime
>>> from django.utils.timezone import get_current_timezone
>>> mydate = datetime.now(tz=get_current_timezone())
>>> mydate
datetime.datetime(2019, 3, 10, 11, 16, 9, 184106, 
    tzinfo=<DstTzInfo 'Europe/Paris' CET+1:00:00 STD>)
>>> str(mydate)
'2019-03-10 11:16:09.184106+01:00'

Another equivalent method is using make_aware, see dmrz post.

Fixed height and width for bootstrap carousel

Because some images could have less than 500px of height, it's better to keep the auto-adjust, so i recommend the following:

<div class="carousel-inner" role="listbox" style="max-width:900px; max-height:600px !important;">`

How to change python version in anaconda spyder

You can open the preferences (multiple options):

  • keyboard shortcut Ctrl + Alt + Shift + P
  • Tools -> Preferences

And depending on the Spyder version you can change the interpreter in the Python interpreter section (Spyder 3.x):

enter image description here

or in the advanced Console section (Spyder 2.x):

enter image description here

Case insensitive regular expression without re.compile?

In imports

import re

In run time processing:

RE_TEST = r'test'
if re.match(RE_TEST, 'TeSt', re.IGNORECASE):

It should be mentioned that not using re.compile is wasteful. Every time the above match method is called, the regular expression will be compiled. This is also faulty practice in other programming languages. The below is the better practice.

In app initialization:

self.RE_TEST = re.compile('test', re.IGNORECASE)

In run time processing:

if self.RE_TEST.match('TeSt'):

How to set iPhone UIView z index?

We can use zPosition in ios

if we have a view named salonDetailView eg : @IBOutlet weak var salonDetailView: UIView!

In my case see the image

and have UIView for GMSMapView eg : @IBOutlet weak var mapViewUI: GMSMapView!

To show the View salonDetailView upper of the mapViewUI

use zPosition as below

salonDetailView.layer.zPosition = 1

How to set underline text on textview?

you're almost there: just don't call toString() on Html.fromHtml() and you get a Spanned Object which will do the job ;)

tvHide.setText(Html.fromHtml("<p><u>Hide post</u></p>"));

__FILE__, __LINE__, and __FUNCTION__ usage in C++

C++20 std::source_location

C++ has finally added a non-macro option, and it will likely dominate at some point in the future when C++20 becomes widespread:

The documentation says:

constexpr const char* function_name() const noexcept;

6 Returns: If this object represents a position in the body of a function, returns an implementation-defined NTBS that should correspond to the function name. Otherwise, returns an empty string.

where NTBS means "Null Terminated Byte String".

I'll give it a try when support arrives to GCC, GCC 9.1.0 with g++-9 -std=c++2a still doesn't support it.

https://en.cppreference.com/w/cpp/utility/source_location claims usage will be like:

#include <iostream>
#include <string_view>
#include <source_location>

void log(std::string_view message,
         const std::source_location& location std::source_location::current()
) {
    std::cout << "info:"
              << location.file_name() << ":"
              << location.line() << ":"
              << location.function_name() << " "
              << message << '\n';
}

int main() {
    log("Hello world!");
}

Possible output:

info:main.cpp:16:main Hello world!

__PRETTY_FUNCTION__ vs __FUNCTION__ vs __func__ vs std::source_location::function_name

Answered at: What's the difference between __PRETTY_FUNCTION__, __FUNCTION__, __func__?

How to disable action bar permanently

try this in your manifist

 <activity
        android:name=".MainActivity"
        android:theme="@android:style/Theme.Holo.NoActionBar"
        android:label="@string/app_name" >

What is a 'multi-part identifier' and why can't it be bound?

I faced this problem and solved it but there is a difference between your and mine code. In spite of I think you can understand what is "the multi-part identifier could not be bound"

When I used this code

 select * from tbTest where email = [email protected]

I faced Multi-part identifier problem

but when I use single quotation for email address It solved

 select * from tbTest where email = '[email protected]'

INSERT with SELECT

I think your INSERT statement is wrong, see correct syntax: http://dev.mysql.com/doc/refman/5.1/en/insert.html

edit: as Andrew already pointed out...

Difference between a virtual function and a pure virtual function

You can actually provide implementations of pure virtual functions in C++. The only difference is all pure virtual functions must be implemented by derived classes before the class can be instantiated.

mysql query: SELECT DISTINCT column1, GROUP BY column2

Try the following:

SELECT DISTINCT(ip), name, COUNT(name) nameCnt, 
time, price, SUM(price) priceSum
FROM tablename 
WHERE time >= $yesterday AND time <$today 
GROUP BY ip, name

How to prevent going back to the previous activity?

Since there are already many great solutions suggested, ill try to give a more dipictive explanation.

How to skip going back to the previous activity?

Remove the previous Activity from Backstack. Simple

How to remove the previous Activity from Backstack?

Call finish() method

The Normal Flow:

enter image description here
All the activities are stored in a Stack known as Backstack.
When you start a new Activity(startActivity(...)) then the new Activity is pushed to top of the stack and when you press back button the Activity is popped from the stack.
One key point to note is that when the back button is pressed then finish(); method is called internally. This is the default behavior of onBackPressed() method.

So if you want to skip Activity B?

ie A<--- C

Just add finish(); method after your startActvity(...) in the Activity B

Intent i = new Intent(this, C.class);
startActivity(i);
finish();

enter image description here

Change background colour for Visual Studio

One line answer, F1 -> search for "Color Theme" -> select the color you like

Change the default editor for files opened in the terminal? (e.g. set it to TextEdit/Coda/Textmate)

For Sublime Text 3:

defaults write com.apple.LaunchServices LSHandlers -array-add '{LSHandlerContentType=public.plain-text;LSHandlerRoleAll=com.sublimetext.3;}'

See Set TextMate as the default text editor on Mac OS X for details.

Appending to an empty DataFrame in Pandas?

You can concat the data in this way:

InfoDF = pd.DataFrame()
tempDF = pd.DataFrame(rows,columns=['id','min_date'])

InfoDF = pd.concat([InfoDF,tempDF])

How to install MinGW-w64 and MSYS2?

Unfortunately, the MinGW-w64 installer you used sometimes has this issue. I myself am not sure about why this happens (I think it has something to do with Sourceforge URL redirection or whatever that the installer currently can't handle properly enough).

Anyways, if you're already planning on using MSYS2, there's no need for that installer.

  1. Download MSYS2 from this page (choose 32 or 64-bit according to what version of Windows you are going to use it on, not what kind of executables you want to build, both versions can build both 32 and 64-bit binaries).

  2. After the install completes, click on the newly created "MSYS2 Shell" option under either MSYS2 64-bit or MSYS2 32-bit in the Start menu. Update MSYS2 according to the wiki (although I just do a pacman -Syu, ignore all errors and close the window and open a new one, this is not recommended and you should do what the wiki page says).

  3. Install a toolchain

    a) for 32-bit:

    pacman -S mingw-w64-i686-gcc
    

    b) for 64-bit:

    pacman -S mingw-w64-x86_64-gcc
    
  4. install any libraries/tools you may need. You can search the repositories by doing

    pacman -Ss name_of_something_i_want_to_install
    

    e.g.

    pacman -Ss gsl
    

    and install using

    pacman -S package_name_of_something_i_want_to_install
    

    e.g.

    pacman -S mingw-w64-x86_64-gsl
    

    and from then on the GSL library is automatically found by your MinGW-w64 64-bit compiler!

  5. Open a MinGW-w64 shell:

    a) To build 32-bit things, open the "MinGW-w64 32-bit Shell"

    b) To build 64-bit things, open the "MinGW-w64 64-bit Shell"

  6. Verify that the compiler is working by doing

    gcc -v
    

If you want to use the toolchains (with installed libraries) outside of the MSYS2 environment, all you need to do is add <MSYS2 root>/mingw32/bin or <MSYS2 root>/mingw64/bin to your PATH.

The provided URI scheme 'https' is invalid; expected 'http'. Parameter name: via

Adding this as an answer, just since you can't do much fancy formatting in comments.
I had the same issue, except I was creating and binding my web service client entirely in code.
Reason is the DLL was being uploaded into a system, which prohibited the use of config files.

Here is the code as it needed to be updated to communicate over SSL...

Public Function GetWebserviceClient() As WebWorker.workerSoapClient
    Dim binding = New BasicHttpBinding()
    binding.Name = "WebWorkerSoap"
    binding.CloseTimeout = TimeSpan.FromMinutes(1)
    binding.OpenTimeout = TimeSpan.FromMinutes(1)
    binding.ReceiveTimeout = TimeSpan.FromMinutes(10)
    binding.SendTimeout = TimeSpan.FromMinutes(1)

    '// HERE'S THE IMPORTANT BIT FOR SSL
    binding.Security.Mode = BasicHttpSecurityMode.Transport

    Dim endpoint = New EndpointAddress("https://myurl/worker.asmx")

    Return New WebWorker.workerSoapClient(binding, endpoint)
End Function

Set left margin for a paragraph in html

<p style="margin-left:5em;">Lorem ipsum dolor sit amet, consectetur adipiscing elit. Ut lacinia vestibulum quam sit amet aliquet. Phasellus tempor nisi eget tellus venenatis tempus. Aliquam dapibus porttitor convallis. Praesent pretium luctus orci, quis ullamcorper lacus lacinia a. Integer eget molestie purus. Vestibulum porta mollis tempus. Class aptent taciti sociosqu ad litora torquent per conubia nostra, per inceptos himenaeos. </p>

That'll do it, there's a few improvements obviously, but that's the basics. And I use 'em' as the measurement, you may want to use other units, like 'px'.

EDIT: What they're describing above is a way of associating groups of styles, or classes, with elements on a web page. You can implement that in a few ways, here's one which may suit you:

In your HTML page, containing the <p> tagged content from your DB add in a new 'style' node and wrap the styles you want to declare in a class like so:

<head>
  <style type="text/css">
    p { margin-left:5em; /* Or another measurement unit, like px */ }
  </style>
</head>
<body>
  <p>Lorem ipsum dolor sit amet, consectetur adipiscing elit. Ut lacinia vestibulum quam sit amet aliquet.</p>
</body>

So above, all <p> elements in your document will have that style rule applied. Perhaps you are pumping your paragraph content into a container of some sort? Try this:

<head>
  <style type="text/css">
    .container p { margin-left:5em; /* Or another measurement unit, like px */ }
  </style>
</head>
<body>
  <div class="container">
    <p>Lorem ipsum dolor sit amet, consectetur adipiscing elit. Ut lacinia vestibulum quam sit amet aliquet.</p>
  </div>
  <p>Vestibulum porta mollis tempus. Class aptent taciti sociosqu ad litora torquent per conubia nostra.</p>
</body>

In the example above, only the <p> element inside the div, whose class name is 'container', will have the styles applied - and not the <p> element outside the container.

In addition to the above, you can collect your styles together and remove the style element from the <head> tag, replacing it with a <link> tag, which points to an external CSS file. This external file is where you'd now put your <p> tag styles. This concept is known as 'seperating content from style' and is considered good practice, and is also an extendible way to create styles, and can help with low maintenance.

Display text on MouseOver for image in html

You can use CSS hover in combination with an image background.

CSS

   .image
{
    background:url(images/back.png);
    height:100px;
    width:100px;
    display: block;
    float:left;
}

.image  a {
    display: none;
}

.image  a:hover {
    display: block;
}

HTML

<div class="image"><a href="#">Text you want on mouseover</a></div>

How do I get the web page contents from a WebView?

Per issue 12987, Blundell's answer crashes (at least on my 2.3 VM). Instead, I intercept a call to console.log with a special prefix:

// intercept calls to console.log
web.setWebChromeClient(new WebChromeClient() {
    public boolean onConsoleMessage(ConsoleMessage cmsg)
    {
        // check secret prefix
        if (cmsg.message().startsWith("MAGIC"))
        {
            String msg = cmsg.message().substring(5); // strip off prefix

            /* process HTML */

            return true;
        }

        return false;
    }
});

// inject the JavaScript on page load
web.setWebViewClient(new WebViewClient() {
    public void onPageFinished(WebView view, String address)
    {
        // have the page spill its guts, with a secret prefix
        view.loadUrl("javascript:console.log('MAGIC'+document.getElementsByTagName('html')[0].innerHTML);");
    }
});

web.loadUrl("http://www.google.com");